#27 Rosh Review

Réussis tes devoirs et examens dès maintenant avec Quizwiz!

Question: What is the treatment for Clamydia trachomatis?

Answer: Azithromycin or Doxycycline.

A patient presents with abdominal bloating and flatulence. She denies diarrhea. Physical examination is significant only for borborygmi. You suspect lactose intolerance and start the patient on lactase supplementation and diet modification. Which of the following supplements should also be prescribed? Calcium Pancreatase Sevelamer Sodium

Correct Answer ( A ) Explanation: Patients with low levels of duodenal lactase are said to have lactose intolerance. This leads to an inability to digest lactose. Although not lethal, this enzyme deficiency can be problematic for up to 25% of the Caucasian population in the US. It is more common in people of Asian, South American and African descent. Milk and processed foods contain lactose, which needs to be hydrolyzed down before absorption can occur. When it is not broken down, large intestinal bacteria ferment it, producing hydrogen gas and increasing intestinal water absorption. This deficiency causes loose stools, bloating, abdominal pain, nausea and flatulence, symptoms which can be confused with inflammatory bowel disease and irritable bowel syndrome. Diagnosis is confirmed with a positive Milk Tolerance Test: give the patient 500 ml of milk, then measure the blood glucose level; an increase of less than 9 mg/dL indicates lactose malabsorption. Loss of lactase does not happen until 5 years of age, so an abnormal breath test in children <5 years of age points to another mucosal problem. Treatment includes dietary modification, especially transitioning to soy based products, and lactase enzyme preparations. When the diet is modified to avoid milk products, supplemental calcium is recommended to prevent osteopenia and osteoporosis. Pancreatase (B) is a pancreas enzyme formulation used for patients with chronic pancreatitis or pancreatic insufficiency, not lactose intolerance. Sevelamer (C) is prescribed to chronic renal failure patients on dialysis with hyperphosphatemia. Sodium supplementation (D) is usually reserved for patients with critical hyponatremia, not lactose intolerance.

Question: Which anti-fibrotic medication has modest benefit in slowing the progression of IPF?

Answer: Pirfenidone. Rapid Review Idiopathic Pulmonary Fibrosis Smoking males Unknown agent → repeated alveolitis → fibrosis Chronic cough, dyspnea Honeycombing Rx: O2, pulmonary rehabilitation

Question: What is the cause of lymphogranuloma venereum?

Answer: Chlamydia trachomatis serovars L1-L3.

Question: What is the most common complication of meningococcemia?

Answer: Myocarditis with congestive heart failure or conduction abnormalities. Rapid Review Meningococcemia Military recruits, students Fever, HA, arthralgias, rash Petechiae, skin lesions with gray necrotic centers Waterhouse-Friderichson syndrome: bilateral adrenal hemorrhage + meningococcemia Ceftriaxone and vancomycin

Question: Is the ototoxicity commonly seen in salicylate toxicity permanent?

Answer: No. The tinnitus, seen at salicylates levels > 20 mg/dL, is reversible.

Question: What is the classic description of ankylosing spondylitis?

Answer: Insidious onset of bilateral buttock pain (sacroiliitis), thoracolumbar pain (enthesitis) and lumbar morning stiffness that lessens with exercise most commonly seen in young men. Rapid Review Back Pain Night pain, weight loss: malignancy Back pain + fever + neurological deficits: epidural abscess Acute bony tenderness: fracture Young, morning stiffness: seronegative spondyloarthropathy Urinary retention: cauda equina syndrome Pain with extension, relief with flexion: spinal stenosis Image if red flags present

Question: What components are contained in cryoprecipitate?

Answer: Factor VIII, XIII, von Willebrand factor (vWF), fibrinogen and fibronectin. Rapid Review Hemophilias X-linked recessive A: Factor VIII deficiency B: Factor IX deficiency Easy bruising, hemarthroses Increased PTT

Question: What is the upper limit of normal for a QTc interval?

Answer: For men, the upper level of QTc interval is 450 msec. For women, the upper level of QTc interval is 470 msec. Rapid Review Hypertrophic Cardiomyopathy Asymmetric LV septal wall hypertrophy → outflow obstruction Autosomal dominant (familial form) Young patient Exertional syncope Sudden cardiac death S4 gallop Midsystolic murmur (↑ as preload ↓) Rx: ßBs or CCBs

Question: Which grading system is used to assess skeletal maturity?

Answer: Risser classification system.

Question: Name two congenital QT-prolongation syndromes?

Answer: Romano-Ward syndrome and Jervell-Lange-Nielsen syndrome. Rapid Review Torsades de Pointes (TdP) ECG will show rhythm > 100 beats per minute and frequent variation in the QRS axis and morphology Most commonly caused by acquired or congenital long QT interval syndrome Treatment is Unstable: defibrillation Stable: intravenous magnesium sulfate and stopping the offending drug

Question: What type of cough is classic of Chlamydia trachomatis pneumonia?

Answer: Staccato cough. Rapid Review Bronchiolitis Patient will be an infant Complaining of difficulty breathing PE will show respiratory distress, polyphonic wheezing, and rales CXR will show diffuse infiltrates Diagnosis is made by history and physical exam Most commonly caused by respiratory syncytial virus (RSV) Treatment is supportive care

Question: Which gender predominates in newly diagnosed cases of Celiac disease?

Answer: Up to 75% of new cases of Celiac disease are diagnosed in women. Rapid Review Celiac Disease Patient will be complaining of diarrhea, steatorrhea, flatulence, weight loss, weakness and abdominal distension Labs will show IgA anti-endomysial (AGA) and anti-tissue transglutaminase (anti-tTG) antibodies Diagnosis is made by small bowel biopsy Treatment is gluten free diet Comments: associated with dermatitis herpetiformis (chronic, very itchy skin rash made up of bumps and blisters)

Question: When should syringe saline irrigation not be used in removing auditory foreign bodies?

Answer: When the object is soft or made of organic matter, like a seed. These objects swell, and further hinder removal Rapid Review Ear Foreign Body Probe, forceps, or irrigation Alive insects: Instill lidocaine to sedate before extraction Examine TM/ear canal after extraction

A 40-year-old woman with a history of triple negative metastatic breast cancer presents with colicky groin pain and hematuria. A CT scan of the abdomen and pelvis is performed. She also reports several days of constipation, weakness, and confusion. Vital signs are as follows: Temp 37°C, HR 90, BP 110/70, RR 12, and oxygen saturation 99% on room air. Physical examination is within normal limits. Serum calcium is 14 mg/dL. Parathyroid hormone is 5 pg/mL. A PTHrP is pending. Besides starting IV normal saline, long-term therapy with which of the following should be initiated? A medication that may cause gray baby syndrome A medication that may cause hot flashes A medication that may cause osteonecrosis of the jaw A medication that may cause pulmonary fibrosis

Correct Answer ( C ) Explanation: Immediate treatment for severe symptomatic hypercalcemia is volume repletion with normal saline. Long-term therapy of hypercalcemia of malignancy involves treatment with bisphosphonates, which may cause osteonecrosis of the jaw. Hypercalcemia of malignancy (driven by PTH related peptide) is most commonly seen as a consequence of squamous cell, renal, breast, or bladder cancer. Hypercalcemia may also be caused by hematologic malignancies via the upregulation of cytokines and vitamin D, as well as via local osteolysis, which is seen in breast cancer and multiple myeloma. In hypercalcemia of malignancy, bisphosphonates are the cornerstone of long-term therapy. Bisphosphonates inhibit osteoclastic activity, reducing both the formation and the resorption of hydroxyapatite. They are used in malignancy-associated hypercalcemia as well as Paget's disease of the bone and postmenopausal osteoporosis. Important toxicities to remember are corrosive esophagitis nausea, diarrhea, and osteonecrosis of the jaw. Chloramphenicol, an antibiotic, may cause gray baby syndrome (A). Drugs such as bleomycin and amiodarone may cause pulmonary fibrosis (C). Tamoxifen and clomiphene may cause hot flashes (B).

Which one of the following screening practices is recommended for the adolescent/young adult population by the U.S. Preventive Services Task Force? Chlamydia screening in sexually active females younger than 25 Lipid screening Scoliosis screening Testicular examination

Correct Answer ( A ) Explanation: Screening for Chlamydia in all sexually active, non-pregnant women <25 years of age is recommended. Chlamydia trachomatis is usually asymptomatic in women, however it may present with purulent discharge on exam as well as a friable cervix. In order to diagnose Chlamydia, a test sample from urine or a cervical swab must be collected for culture. Lipid Screening (B) is recommended in adult men 35 years or older and 45 years or older in adult women. It is recommended that screening be performed with a fasting lipid profile or non-fasting total and HDL cholesterol. There is no current recommendation on routine screening for scoliosis (C). A routine testicular examination (D) is performed in infants in order to verify descent of both testes, however there is no need for routine testicular examination in adolescents unless there is a warranted cause such as an infection, torsion, varicocele or a hernia. Routine screening for testicular cancer is not recommended.

An 18-year-old man presents after a syncopal episode. The patient states he was running when he suddenly felt lightheaded and nauseated leading to him passing out. On presentation, he is asymptomatic with normal vital signs. Examination reveals a healthy man with a crescendo-decrescendo murmur heard at the left sternal border. What diagnosis should be suspected in this patient? Hypertrophic cardiomyopathy Hypovolemic state Vasovagal syncope Wolff-Parkinson-White syndrome

Correct Answer ( A ) Explanation: This 18-year-old man has exertional syncope raising the suspicion of hypertrophic cardiomyopathy (HCM). HCM is an autosomal dominant genetic abnormality in which the left ventricle (particularly the septum more than the free wall) is hypertrophied. The typical presentation is dyspnea on exertion in a young patient with no other cardiac disease. About 20-30% will present after a syncopal event and many present with sudden death. Physical examination may reveal a loud S4 gallop and a harsh mid-systolic crescendo-decrescendo murmur. The murmur can be increased by maneuvers that decrease left ventricular filling (standing or performing a Valsalva maneuver) and decreased by maneuvers that increase left ventricular filling (squatting, Trendelenberg). ECG is abnormal in 90% of patients and the classic ECG finding is left ventricular hypertrophy with deep, narrow Q waves particularly in the precordial leads. Diagnosis can be made by echocardiography but cardiac catheterization is the gold standard. Vasovagal syncope (C) is an autonomic response in which the patient experiences bradycardia and hypotension mediated by the vagus nerve in response to stress or pain. This is the most common form of syncope but is not associated with a pathologic murmur or exertion. The patient may be at risk of being hypovolemic (B) from dehydration as he was running but it is unlikely for a patient to have a syncopal episode due to hypovolemia and present with normal vital signs. Wolff-Parkinson-White (WPW) syndrome (D) is a disorder in which the patient has an accessory tract between the atrium and the ventricle predisposing them to develop reentrant tachycardias. The typical ECG findings in WPW are a shortened PR interval (< 0.12 seconds), a delta wave (upsloping deflection into the QRS complex) and a widened QRS duration (>0.10 seconds).

An anxious and agitated 3-year-old boy presents with hearing loss and otalgia. Examination reveals a cockroach deeply lodged in the external auditory canal. You prepare for forceps removal, but the child cannot remain still and calm enough to safely proceed. Which of the following procedures can be safely used in this situation? Bulb syringe irrigation Mineral oil instillation Myringotomy Stapedectomy

Correct Answer ( B ) Explanation: It is common for children to place foreign bodies into their ears. Insects, commonly cockroaches, can also enter and become lodged in the external auditory canal. Pathology includes epithelial trauma with resultant inflammation, tympanic membrane disruption with conductive hearing loss and oval or round window injury with sensorineural hearing loss. Most foreign bodies can be removed with forceps. An alternative method includes bulb syringe saline irrigation. However, in anxious children, further damage can be attributed to both of these methods. With firmly embedded objects, local anesthetic may be required to safely remove the object. When managing a lodged insect in an anxious child, a viable method, which lessens iatrogenic trauma, involves insect suffocation with mineral oil instillation into the canal. Bulb syringe irrigation (A) is a commonly used foreign body removal method, but should be reserved for patients who can safely stay still, as its timeliness and insertion depth can cause further auditory damage in an agitated individual. Myringotomy (C), an incision in the tympanic membrane, is used to drain suppurative otitis media. It is not a foreign body removal method. Stapedectomy (D) is a procedure used to correct conductive hearing loss secondary to otosclerosis. It is not a procedure used to remove a foreign body.

A 17-year-old boy is concerned that he contracted a sexually transmitted infection from an episode of unprotected sexual intercourse during a trip to South Africa. Symptoms began with a very painful papule on his penis. The papule gradually ulcerated, and its edges became irregular. One week later, he developed painful right-sided inguinal adenopathy. Which empirical antibiotic is most likely to treat his infection? Acyclovir Azithromycin Benzathine penicillin G Trimethoprim-sulfamethoxazole

Correct Answer ( B ) Explanation: The patient has signs and symptoms of chancroid, a sexually transmitted infection caused by Haemophilus ducreyi. The infection is most common in developing and third world countries. Following an incubation period of 3-10 days, infected patients develop a painful genital papule or pustule, which then ulcerates to leave a ragged edge surrounding a necrotic center. Within one to two weeks, approximately half of affected patients develop tender, unilateral inguinal adenopathy, which may suppurate and form a fistula to the skin. Haemophilus ducreyi can be isolated from a swab of the ulcer or an aspirate of a fluctuant or draining node. Treatment options include azithromycin, erythromycin, ceftriaxone, or ciprofloxacin and, if possible, incision and drainage of suppurative lymph nodes. Acyclovir (A) is used for treatment of genital herpes infection, which presents with multiple painful vesicles and ulcerations and tender bilateral inguinal adenopathy. Benzathine penicillin G (C) treats early syphilis, which is characterized by a chancre, a painless papule with a shallow central ulceration. Trimethoprim-sulfamethoxazole (D) is appropriate treatment for granuloma inguinale, a sexually transmitted infection that is caused by Klebsiella granulomatis. The painless genital ulcer of granuloma inguinale has a rolled, distinct border with a beefy red, clean base, and ulcer biopsy contains classic Donovan bodies. Granuloma inguinale is more common in India and New Guinea.

A 3-month-old infant is transferred from an outside hospital with three days of cough and congestion. Today, she developed increased work of breathing. Exam was significant for intercostal retractions, diffuse soft expiratory wheezing, and coarse crackles. A chest radiograph shows bilateral hyperinflation and areas of atelectasis with volume loss. What is the most likely diagnosis? Bacterial pneumonia Status asthmaticus Viral bronchiolitis Viral pneumonia

Correct Answer ( C ) Explanation: Bronchiolitis is a clinical syndrome resulting from inflammation of the bronchioles. It is extremely common during the fall and winter, and it occurs almost exclusively in infants and children < 2 years of age. The most common viral etiology is Respiratory Syncytial Virus (RSV), but it may also be caused by parainfluenza, rhinovirus, human metapneumovirus, influenza, human bocavirus, and adenovirus. Symptoms typically begin with upper respiratory congestion and progress to cough and other lower respiratory symptoms, such as chest congestion, tachypnea, and increased work of breathing. Bronchiolitis is a self-limited process, but the associated cough may last up to four weeks. Some children will require hospitalization for supplemental oxygen, high flow nasal cannula or mechanical ventilation, or intravenous hydration. Infants and children without respiratory distress, hypoxemia, or dehydration may be treated in the outpatient setting with management of secretions and monitoring of fluid status. Bacterial pneumonia (A) and viral pneumonia (D) are less likely than bronchiolitis in the above patient. Viruses are the most common etiology of pneumonia in infants. Chlamydia trachomatis may also cause classic afebrile interstitial pneumonia in this age group, but symptoms are almost always present by 8 weeks of life. In the above patient, the chest radiograph is not consistent with lobar or interstitial pneumonia. Notably, chest radiographs are not indicated in bronchiolitis. Status asthmaticus (B) is not diagnosed in infants. In general, asthma is not diagnosed in a child < 2 years of age. Infants and children with bronchiolitis may wheeze and are at increased risk of developing asthma in the future. However, it is unclear whether asthma is caused by previous respiratory infections or if wheezing with bronchiolitis indicates an underlying propensity toward asthma. Notably, infants and children with bronchiolitis do not benefit from the same treatments as children with status asthmaticus, and as such, short-acting bronchodilators or steroids are not recommended in the 2014 AAP Bronchiolitis Guidelines.

A 25-year-old carpenter presents to your primary care clinic with acute, atraumatic low back pain. This is his first episode of back pain. He denies weakness, numbness, fever or bowel and bladder incontinence. Which of the following pain descriptors, gathered from a detailed history and examination, would most prompt you to order imaging on this initial visit? Axial pain Flexion-based pain Night pain Pain began after lifting a heavy object

Correct Answer ( C ) Explanation: The evaluation of back pain begins with assessing for "red flags," such as night pain and weight loss (suspicious for tumor), fever, chills, and sweats (consider infection), acute bony tenderness (consider fracture), morning stiffness lasting > 30 minutes in young adults (consider seronegative spondyloarthropathy, and any neurologic deficit or bowel or bladder involvement (consider nerve root compromise). Acute lumbar sprain is usually the result of repeated lifting or twisting, typically occurring in young men with laborious vocation. Axial lumbar pain is typical of sprain and internal disk disruption. Radicular pain is typical of disk bulge or herniation with nerve root encroachment. Flexion-based axial lumbar pain is typical of disk pathology, whereas extension-based axial lumbar pain suggests facet joints, central stenosis or foraminal stenosis are the main pain generators. Imaging plays a limited role in acute sprain, however, it should be considered for acute lumbar pain associated with any neurologic abnormality, trauma, concerning past medical or family history, or any case involved with fever, sweats, chills, weight loss, night pain or pain at rest. Axial pain (A), flexion-based pain (B), pain began after lifting a heavy object (D) are typical of acute lumbar sprain. Acute, flexion-based, axial lumbar pain is typical of disk tear, bulge or herniation. However, a radiograph in the setting of absent "red flags", is low yielding and not recommended at the initial visit.

A patient with palpitations presents to the ED. Her rhythm strip is seen above. Which of the following is the most appropriate initial management? Amiodarone Cardioversion Magnesium sulfate Transvenous pacing at 60-80 bpm

Correct Answer ( C ) Explanation: Torsades de pointes is a form of polymorphic ventricular tachycardia. It is characterized by a fluctuating amplitude of the QRS complexes which appear to twist around the isoelectric line. Torsades is associated with prolonged QT syndrome, hypokalemia and hypomagnesemia. It can deteriorate into ventricular fibrillation. Symptoms include palpitations, dizziness, syncope and sudden death. Acute management begins with intravenous magnesium. Amiodarone (A) prolongs the QT interval. As such, it is contraindicated in torsades de pointes. Cardioversion (B) is a last resort treatment of torsades due to the fact that many cases of torsades are self-limiting and improve with administration of magnesium. A transvenous pacer (D) is typically used to treat bradydysrhythmias. It also can be used to overdrive pace the ventricles which may lead to cessation of torsades. However, inserting a transvenous pacer is time consuming and the initial treatment of torsades is intravenous magnesium.

A patient with hemophilia B presents with hemarthrosis of the left elbow. Which of the following is the most appropriate management indicated? Cryoprecipitate Desmopressin Factor VIII concentrate Recombinant factor IX

Correct Answer ( D ) Explanation: This patient presents with hemarthrosis and a history of hemophilia B or factor IX deficiency requiring treatment with recombinant factor IX. Hemophilia B or Christmas disease is an X-linked recessive bleeding disorder. The genetic alteration results in a deficiency of factor IX activity. It is genetically and clinically indistinguishable from hemophilia A but is far less common. The most common clinical presentation is hemarthrosis with minor trauma but internal bleeding with minor head, chest or abdominal trauma can be seen as well. Historically, treatment involved the use of fresh frozen plasma. More recently, plasma prothrombin complex (factors II, VII, IX and X) and recombinant factor IX have become available and have less side effects. If available, recombinant factor IX is the preferred treatment. Cryoprecipitate (A) does not possess factor IX and is therefore not indicated in the treatment of bleeding in Hemophilia B. Desmopressin (B) stimulates release of von Willebrand factor from endothelial cells is similarly ineffective in management of this disorder. Factor VIII concentrate (C) is useful in the treatment of Hemophilia A.

A previously healthy 13-year-old girl presents to your office after a positive scoliosis screening at school. X-ray results reveal a Cobb angle of 15 degrees. Which of the following is the most appropriate next step in management? Bracing Reassurance that no follow-up is necessary Surgical referral Watchful waiting and 6 month follow-up

Correct Answer ( D ) Explanation: Scoliosis is a lateral curvature of the spine. Idiopathic scoliosis is the most common type of scoliosis seen by orthopedic surgeons and often presents in adolescence. Adolescent idiopathic scoliosis is defined as a Cobb angle greater than or equal to 10 degrees, age of onset 10 years or greater and no identifiable etiology. The Cobb angle is determined on radiography and is the measurement used to monitor the progression of scoliosis. As patients mature, skeletal curves can progress. Assessment of risk factors for progression affects management, however completely accurate prediction is impossible. Risk factors include age less than 12 years, onset prior to menarche, curves with a Cobb angle greater than or equal to 20 degrees, female sex, and double or thoracic curves. Cobb angle measurements also guide management of patients with scoliosis. Patients with a Cobb angle of 10-19° should be observed and followed clinically every 6-9 months until they reach skeletal maturity. Bracing (A) is recommended for patients with a Cobb angle of 30-39°, although some patients and families will choose bracing with a Cobb angle of 20-29°. Reassurance that no follow up is necessary (B) is correct for patients with a Cobb angle less than 10°, as this does not meet the definition for scoliosis. Surgical referral (D) is indicated for patients with a Cobb angle greater than or equal to 40°.

Which of the following is most appropriate in the treatment plan for a patient with idiopathic pulmonary fibrosis? Albuterol Azathioprine Bosentan Pulmonary rehabilitation

Correct Answer ( D ) Explanation: The goal in the treatment of idiopathic pulmonary fibrosis is to reduce symptoms, prevent acute exacerbations, vaccinate, slow progression and increase survival. There are several treatment options, however, most medication trials have shown no benefit. Sufficient clinical evidence that any treatment improves survival or quality of life for patients with idiopathic pulmonary fibrosis (IPF) is lacking. Supportive care, supplemental oxygen and pulmonary rehabilitation appear to be the most beneficial options. Lung transplantation may be an option in those with disease progression and minimal comorbidities. IPF prognosis is poor. Only 20-30% of patients survive five years after diagnosis. There is no role for bronchodilators in IPF as the pathology is not bronchoconstriction but is fibrotic restriction. Albuterol's (A) bronchodilatory action cannot reverse this fibrosis. Classically, IPF was treated with a triple therapy consisting of N-Acetlycysteine, prednisone and azathioprine (B). However, a recent study was ended early due to increased deaths and hospitalizations in patients receiving this treatment and N-Acetlycysteine alone. As such, neither the triple therapy nor N-Acetlycysteine alone is strongly recommended for the treatment of IPF. Bosentan (C) is no longer considered an effective treatment for IPF. It is however used in the treatment of pulmonary arterial hypertension due to its beneficial action of endothelin-receptor antagonism.

A 72-year-old woman presents to the ED with an acute onset of dyspnea and palpitations that began four hours prior to arrival. Vital signs include a heart rate of 144 beats per minute, blood pressure of 80/50 mm Hg, respiratory rate of 28 breaths per minute, temperature of 37.0°C, and a pulse oximetry of 88% on room air. The above 12-lead ECG is obtained. What is the most appropriate next step in the management of this patient? Anticoagulation with enoxaparin followed by warfarin Chemical cardioversion using procainamide Rate control using diltiazem Rate control using esmolol Synchronized cardioversion

Correct Answer ( E ) Explanation: This patient has atrial fibrillation with a rapid ventricular rate and is hemodynamically unstable (BP 80/50 mm Hg and pulse ox 88%). This dysrhythmia needs to be emergently corrected in order to stabilize the patient; failure to do so could result in sudden cardiac death. In such circumstances, emergent synchronized cardioversion is required. If the patient is hemodynamically stable, the approach to management (rate vs. rhythm control) depends on the time of arrhythmia onset. If the onset is unknown or greater than 48 hours, then cardioversion should be delayed until the patient can be adequately anticoagulated with enoxaparin (A) followed by warfarin. In the interim, rate control would be accomplished with either esmolol (D), a short acting ß-blocker, or diltiazem (C), a calcium channel blocker. For patients with stable paroxysmal atrial fibrillation and duration of onset < 48 hours, chemical cardioversion using procainamide (B) can be attempted. In unstable patients, regardless of the rhythm duration, synchronized electrical cardioversion is recommended. Alternatives to procainamide include amiodarone, ibutilide, and, to a lesser degree, flecainide.

Question: Medications in which the fetal risks clearly outweigh the benefits are considered to be in what category?

Answer: Category X. Rapid Review FDA Pregnancy Categories A: No risk in controlled human studies B. No risk in controlled animal studies C: Small risk in controlled animal studies D. Strong evidence of risk to fetus X: Very high risk to fetus

Question: What is the treatment of choice for children with developmental dysplasia of the hip whom are older than 6 months of age?

Answer: Closed reduction.

Question: What is the differential diagnosis for non-traumatic subungual hematoma?

Answer: Melanoma and Kaposi sarcoma. Rapid Review Trephination, Nail Indication: subungual hematoma >50% of nail bed surface or smaller hematomas if they are painful Methods: handheld electrocautery (preferred), heated paper clip, 18-gauge needle/syringe Post procedure care: keep clean and dry for 2 days

Question: Name one opioid antagonist used in the treatment of constipation?

Answer: Methylnaltrexone (Relistor®).

Question: What is the most common complication following local injection of lidocaine?

Answer: Pain. Rapid Review Local Anesthetics Amides: lidocaine, bupivacaine (2 Is) Esters: tetracaine, benzocaine (1 I) Esters: allergenic 2º to (PABA) Benzocaine: methemoglobinemia Lidocaine: seizures, hypotension Bupivicaine: cardiotoxicity Concentration 1% = 1g/100mL = 10 mg/mL Toxicity rx: lipid emulsion

Question: Acetazolamide is contraindicated in which patients?

Answer: Patients with sickle cell disease and sulfonamide allergies. Rapid Review Acute Mountain Sickness Patient will be climbing a mountain Complaining of "Hangover" like symptoms, headache, nausea, vomiting, insomnia Treatment is halt ascent, acetazolamide Comments: Sulfa allergy - avoid acetazolamide

Question: What are the classic neuropathologic changes found in patients with cerebral palsy?

Answer: Periventricular leukomalacia and matrix hemorrhages. Rapid Review Cerebral Palsy Nonprogr​essive CNS injury Risk factors: substance abuse, low birth weight Most common form: limb spasticity

Question: Which parasitic infection is associated with esophageal varices?

Answer: Schistosomiasis (trematode infection). Rapid Review Esophageal Varices Dilated submucosal left gastric veins Cirrhosis → portal HTN → varices Most common cause of death in cirrhosis Rupture → massive hematemesis Treatment: Ppx: BBs Somatostatin Active bleed: banding, balloon tamponade (last resort) TIPS

Question: Pasteurella grows readily in which medium?

Answer: Sheep blood agar. Rapid Review Cat Bite Most commonly caused by Pasteurella multocida Treatment is irrigate, leave wound open, amoxicillin - clavulanate Complications: Osteomyelitis, Tenosynovitis

Question: Which bacterial infection commonly occurs prior to the development of Guillain-Barré syndrome?

Answer: Campylobacter jejuni infection. Rapid Review Guillain-Barré Syndrome Antecedent pulmonary or GI illness (Campylobacter jejuni) Rapid ascending symmetrical weakness Lower extremity weakness > upper extremity weakness Deep tendon reflex loss → respiratory failure Normal rectal tone CSF: markedly ↑ protein with up to 100 lymphocytes/μL Obtain pulmonary function tests Rx: IVIG or plasmapharesis, possible prophylactic intubation

Question: Most primary endometrial carcinomas are of which histological form?

Answer: Adenocarcinoma. Rapid Review Endometrial Cancer Patient will be a postmenopausal woman Complaining of abnormal vaginal bleeding Diagnosis is made by transvaginal ultrasound or endometrial biopsy Most common type is adenocarcinoma Treatment is total abdominal hysterectomy and bilateral salpingo-oophorectomy (TAH-BSO)

Question: A patient with conduct disorder diagnosed at 15 years of age may go on to develop which personality disorder as an adult?

Answer: Antisocial personality disorder. Rapid Review Conduct Disorder Pediatric version of antisocial personality disorder Violating human rights of others

Question: What pulmonary finding is pathognomonic for sarcoidosis?

Answer: Bilateral hilar adenopathy.

Question: Adolescent females taking injectable depomedroxyprogesterone acetate (Depo-Provera®) for a duration of 2 or more years should be monitored for which side effect?

Answer: Bone mineral density loss.

Question: What medication is relatively contraindicated in hydrocarbon ingestion?

Answer: Epinephrine, due to the possibility of inducing ventricular fibrillation.

Question: What condition causes lower back pain that is worse with walking but improves with rest and bending forward?

Answer: Spinal stenosis. Rapid Review Cauda Equina Syndrome Patient will be someone with a history of trauma, malignancy, epidural abscess, or hematoma Complaining of acute onset of lower back pain with weakness and numbness PE will show urinary retention, saddle anesthesia, decreased rectal tone Diagnosis is made by MRI or CT myelogram Most commonly caused by a herniated disc Treatment is operative decompression

Question: What radiographic sign is present in both croup and bacterial tracheitis?

Answer: Steeple sign. Rapid Review Bacterial Tracheitis Patient will be 3 - 5 years old Complaining of high fever, barky cough, stridor Treat as airway emergency! Comments: Patient will look like croup but toxic appearing

Question: What is the gold standard diagnostic test for spontaneous pneumomediastinum?

Answer: The CT scan. Rapid Review Pneumomediastinum Spontaneous, trauma, Valsalva (crack users), Boerhaave syndrome (hx of vomiting) PE: crepitus, Hamman sign CXR: ring around right pulmonary artery Usually self-resolving

Question: What classification system is commonly used to quantify upper airway narrowing?

Answer: The modified Mallampati classification. Rapid Review Obstructive Sleep Apnea Patient will be obese With a history of allergies Complaining of apneic episodes while sleeping, snoring and daytime sleepiness PE will show enlarged tonsils Diagnosis is made by sleep study Treatment is weight loss, CPAP Comments: can lead to pulmonary hypertension and cor pulmonale (right ventricular hypertrophy)

Question: What is the initial treatment for hypercalcemia?

Answer: Volume repletion and loop diuretics.

A 26-year old woman presents to urgent care with a fever of 101.4°F, dysuria and nausea for the past 24 hours. On physical exam, exquisite pain is elicited on palpation of the right flank. She does not appear toxic. She is able to eat and drink, despite her nausea. Her pregnancy test is negative and her urinalysis is pending. What is the best management and treatment for this patient? Inpatient management with intravenous ceftriaxone Inpatient management with intravenous vancomycin Outpatient management with oral ciprofloxacin Outpatient management with oral trimethoprim/sulfamethoxazole

Correct Answer ( C ) Explanation: Acute pyelonephritis is a common bacterial infection of the renal pelvis and kidney most often seen in young adult women. Most patients have fever, although it may be absent early in the illness. Flank pain is nearly universal, and its absence should raise suspicion of an alternative diagnosis. Risk factors for acute pyelonephritis in nonpregnant women include sexual intercourse three or more times per week during the previous 30 days, urinary tract infections in the previous 12 months, diabetes, stress incontinence in the previous 30 days, a new sex partner in the previous year, recent spermicide use, and a history of urinary tract infections in the patient's mother. A positive urinalysis confirms the diagnosis in patients with a compatible history and physical examination. Urine culture should be obtained in all patients to guide antibiotic therapy if the patient does not respond to initial empiric antibiotic regimens. Escherichia coli is the most common pathogen in acute pyelonephritis, and in the past decade, there has been an increasing rate of E. coli resistance to extended-spectrum beta-lactam antibiotics. Uncomplicated acute pyelonephritis typically occurs in healthy, young women without structural or functional urinary tract abnormalities and without relevant comorbidities. Complicated acute pyelonephritis occurs in patients with a structurally or functionally abnormal genitourinary tract, or a predisposing medical condition. Most cases of uncomplicated acute pyelonephritis can be managed in the outpatient setting. However, patients who appear ill may have severe pyelonephritis or a complication of acute pyelonephritis and should be considered for hospitalization and further evaluation. For uncomplicated pyelonephritis, outpatient treatment with fluoroquinolones is the preferred empiric antimicrobial class in communities where the local prevalence of resistance of community-acquired E. coli is 10 percent or less. If the prevalence of fluoroquinolone resistance among relevant organisms does not exceed 10 percent, patients not requiring hospitalization can be treated with oral ciprofloxacin (500 mg twice per day for seven days), or a once-daily oral fluoroquinolone, such as ciprofloxacin (1,000 mg, extended-release, for seven days) or levofloxacin (750 mg for five days). If the resistance rate exceeds 10 percent, an initial intravenous dose of ceftriaxone or gentamicin should be given, followed by an oral fluoroquinolone regimen. Due to increased resistance, outpatient management with oral trimethoprim/sulfamethoxazole (D) is usually reserved for cases where susceptibility results for the urine isolate are known and indicate likely activity. For women with complicated acute pyelonephritis, inpatient therapy with intravenous ceftriaxone (A) is recommended. Other options for inpatient management with intravenous antibiotics include a fluoroquinolone, an aminoglycoside, an extended-spectrum penicillin, or a carbapenem. Inpatient treatment with intravenous vancomycin (B) is not recommended because this antibiotic does not have gram-negative coverage.

2-week old infant is brought to the office for a well child visit. The physical examination is completely normal except for a clunking sensation and feeling of movement when adducting the hip and applying posterior pressure. Which one of the following would be the most appropriate next step? Plain radiograph of the pelvis Reassurance that the problem resolves spontaneously in 90% of cases, and follow-up in 2 weeks Referral for orthopedic consultation Triple diapering and follow-up in 2 weeks

Correct Answer ( C ) Explanation: Developmental dysplasia of the hip is a common musculoskeletal condition in newborns. Developmental dysplasia of the hip encompasses both subluxation and dislocation of the newborn hip, as well as anatomic abnormalities. It is more common in firstborns, females, breech presentations, oligohydramnios, and patients with a family history of developmental dysplasia. Physical examination screening is recommended by the American Academy of Pediatrics and the Pediatric Orthopaedic Society of North America. The Ortolani, reducing a dislocated hip, and the Barlow, dislocating an unstable hip, maneuvers are the physical examination tests most commonly performed for detection of developmental dysplasia in early infancy. The American Academy of Pediatrics recommends ultrasound screening at six weeks for breech girls, breech boys (optional), and girls with a positive family history of developmental dysplasia of the hip. Infants with developmental hip dyplasia, whether treated or untreated, have a higher incidence of early-onset hip osteoarthritis in adulthood, but the incidence is lowest in those who receive early treatment. Experts are divided with regard to whether hip subluxation can be merely observed during the newborn period, but if there is any question of a hip problem on examination by 2 weeks of age, the recommendation is to refer to a specialist for further testing and treatment. Closed reduction and immobilization in a Pavlik harness, with ultrasonography of the hip to ensure proper positioning, is the treatment of choice until 6 months of age. The patient in the above scenario should be referred for orthopedic consultation. Plain radiographs (A) may be helpful after 4-6 months of age, but prior to that time the ossification centers are too immature to be viewed by plain radiographs. Ultrasound is the imaging study of choice for infants before 4 months of age. Reassurance and follow-up in 2 weeks (B) is not recommended at this point. If these physical findings were found at birth then the recommendation would be to follow-up in 2 weeks. According to Barlow's study, more than 60% of newborns with hip instability became stable by age 1 week, and 88% became stable by age 2 months. Triple diapering (D) should not be used because it puts the hip joint in the wrong position and may aggravate the problem.

A new nursing home employee is being screened for tuberculosis and develops a 6 mm induration on his right volar forearm after injection of a purified protein derivative (PPD). This reading indicates prior tuberculosis exposure if he has had which of the following life circumstances? He is a former guard at a correctional facility He is a HIV-negative injection drug user He is a recent immigrant from Latin America He is an organ recipient on daily immunosuppressives

Correct Answer ( D ) Explanation: An organ recipient on daily immunosuppression therapy is among the group of people who would be considered to have a positive purified protein derivative tuberculin skin test, or PPD, at a reading of 5 mm or greater of induration. Other patients considered to have a positive PPD at 5 mm would include those with recent contact to people with active tuberculosis, those with chest radiographs suggestive of infection, and any HIV-positive patients. Patients who recently emigrated from countries with a high tuberculosis prevalence, are injection drug users, or are residents or employees of high-congregate areas are considered to have a positive PPD reading at 10 mm. Many immunocompromising conditions also allow for a positive reading at 10 mm. Otherwise healthy adults without significant risk of prior exposure would have a positive PPD read at 15 mm of induration. The tuberculin skin test, in which purified protein derivative is placed under the skin of the volar forearm, is the most common method used in screening at-risk people for tuberculosis exposure. This test has high sensitivity and specificity. However, patients who were previously vaccinated with bacillus Calmette-Géurin may falsely test positive. In these patients, an interferon gamma release assay should replace the PPD for tuberculosis screening. All suspected cases of tuberculosis should be reported to government health authorities. A sputum culture should be ordered to determine drug sensitivities of the isolate. Beyond routine screenings, clinicians should be alert for signs and symptoms of pulmonary tuberculosis in patients. These include persistent cough, anorexia, weight loss, fever, night sweats, and blood-streaked sputum production. If tuberculosis is diagnosed, patients must complete treatment with a 6-9 month course of an appropriate anti-tuberculosis drug regimen to prevent the spread of illness. A guard at a correctional facility (A), an HIV-negative injection drug user (B), and a recent immigrant from Latin America (C) would all need a PPD reading of 10 mm of induration or greater for there to be concern of a previous exposure to tuberculosis.

Question: Which disease is responsible for over 95% of cases of hypothyroidism?

Answer: Primary thyroid disease.

Question: What is a complication of cluster headaches?

Answer: Horner syndrome. The severity of pain is also associated with increased risk for suicide. Rapid Review Cluster Headache Patient will be a man Complaining of sudden onset, unilateral, and repetitive brief HAs PE will show ipsilateral conjunctival injection, lacrimation, and rhinnorhea Treatment is Acute: High flow O2; Prophylaxis: CCB's

Question: What unique GI finding is associated with Kawasaki disease?

Answer: Hydrops of the gallbladder; which is characterized by RUQ pain, jaundice, and boggy gallbladder on abdominal ultrasound. Rapid Review Kawasaki Disease Patient will be a child < 4 years old With a history of high fever for 5 days Complaining of conjunctivitis, rash, adenopathy, strawberry tongue, hand/feet edema, fever Treatment is IVIG + aspirin Comments: #1 cause of pediatric acquired heart disease, risk for coronary artery aneurysm Mnemonic: CRASH and burn: Conjunctivitis, Rash, Adenopathy, Strawberry tongue, Hand/feet edema, Fever

Question: Which pathological disorder is associated with pretibial myxedema?

Answer: Hyperthyroidism, specifically Grave's disease.

Question: What are the two major side effects of procainamide?

Answer: Hypotension and prolonged QT interval. Rapid Review Ventricular Tachycardia > 3 consecutive ectopic ventricular beats Monomorphic, polymorphic Bidirectional: digoxin toxicity Wide complexes Pulseless: immediate defibrillation Unstable: synchronized cardioversion Stable: procainamide, amiodarone, synchronized cardioversion (refractory)

Question: What is the management and treatment for pyelonephritis in pregnancy?

Answer: Inpatient management with an intravenous second- or third-generation cephalosporin. Rapid Review Acute Pyelonephritis Patient will be complaining of fever, dysuria, and flank pain PE will show CVA tenderness Urinalysis will show leukocyte esterase, nitrites and microscopy will show WBCs. Most commonly caused by Escherichia coli Treatment is ciprofloxacin Inpatient/pregnant: ampicillin/gentamycin or 3rd gen cephalosporin

Question: What are four common medications included in the regimen approved for the treatment of pulmonary tuberculosis?

Answer: Isoniazid, ethambutol, pyrazinamide, and rifampin are effective anti-tuberculosis drugs, with treatment courses ranging from 6-9 months depending on individual patient circumstances.

Question: Why is ovarian torsion more common on the right side?

Answer: It is believed that the sigmoid colon stabilizes the left ovary so that it is less likely to torse. Rapid Review Ovarian Torsion Patient will be a woman age 15 - 30 or postmenopausal Complaining of sudden onset of unilateral (right > left) abdominal and pelvic pain Labs will show leukocytosis Diagnosis is made by TVUS. Gold standard: Laparoscopy Most commonly caused by cyst or tumor Treatment is emergent surgery

Question: Which reaction may be seen following initiation of antibiotic therapy for Lyme disease?

Answer: Jarisch-Herxheimer reaction.

Question: What is the most common presentation of acute lymphocytic leukemia (ALL)?

Answer: Lymphadenopathy. Rapid Review Acute Lymphocytic Leukemia (ALL) Patient will be a child Complaining of limping, bone pain, fever, and lymphadenopathy PE will show hepatosplenomegaly Labs will show anemia, thrombocytopenia and lymphoblasts on peripheral smear Comments: Most common acute leukemia in children

Question: What is the main difference between posttraumatic stress disorder and adjustment disorder?

Answer: Posttraumatic stress disorder is associated with re-experiencing and avoidance of the stressful event and increased arousal. Rapid Review Post-Traumatic Stress Disorder (PTSD) Sx duration > 1 month Persistent re-experiencing of event Persistently ↑ arousal Avoidance of stimuli ↑ Risk for suicide, substance abuse

Question: What are the three contraindications to measles vaccination?

Answer: Pregnancy, immunosuppresssion, and allergy. Rapid Review Measles (Rubeola) Patient will be an unvaccinated young child With a history of a maculopapular rash that started on head and spreads towards feet Complaining of high fever, cough, conjunctivitis, coryza PE will show red spots with blue/white center on buccal mucosa (Koplik spots) Diagnosis is made clinically Treatment is supportive care

Question: What is borborygmi?

Answer: Common in patients with lactose intolerance, borborygmi refers to the gurgling or rumbling sound which emanates from the stomach when peristaltic waves churn up food, fluid and gas. Rapid Review Lactose Intolerance Genetic condition or 2° to underlying conditions Dx: usually clinical, lactose tolerance test, lactose breath hydrogen test Rx; lactose restriction, calcium, vitamin D

Question: What is the treatment of multiple myeloma?

Answer: Corticosteroids, chemotherapeutics (melphalan, bortezomib, doxorubicin), interferons, and bisphosphonates (pamidronate). Rapid Review Multiple Myeloma Patient will be elderly Complaining of back pain PE will show CRAB: hyperCalcemia, Renal insufficiency, Anemia, lytic Bone lesions/Back pain Labs will show monoclonal antibody spike X-ray will show lytic lesions Peripheral blood smear: Rouleaux formations Serum protein electrophoresis: M spike Protein electrophoresis urine analysis: Bence-Jones proteins Most commonly caused by single clone plasma cell malignancy

Question: What is the causative organism of hand-foot-and-mouth disease?

Answer: Coxsackie virus. Rapid Review Auto-Eczematization (ID Reaction) Patient with a history of a recent fungal infection Complaining of erythematous, papular rash over palms and fingers Most commonly caused by delayed-type hypersensitivity reaction Treatment is resolution of the primary infection

Question: What is the skin finding in methmoglobinemia?

Answer: Cyanotic skin. Rapid Review Carbon Monoxide (CO) Poisoning: Patient will be complaining of headache, nausea, and dizziness PE will show cherry-red skin Labs will show carboxyhemoglobin Treatment is 100% oxygen, hyperbaric oxygen

Question: Which medications are associated with periodic sinus arrest?

Answer: Digitalis, procainamide and quinidine. Rapid Review Sick Sinus Syndrome (SSS) SA node dysfunction Tachycardia-bradycardia syndrome: sinus rate varies from fast to slow and back again Syncope, palpitations Definitive rx: pacemaker placement + rate control medication Untreated SSS → sinus block or sinus arrest

Question: What is the primary treatment for Lyme disease?

Answer: Doxycycline. Rapid Review Bell's Palsy Patient with a history of viral prodrome Complaining of waking up with unilateral facial nerve paralysis, hyperacusis and taste disturbance PE will show CN VII nerve palsy that does not spare the forehead Most commonly caused by HSV Treatment is prednisone, artificial tears, tape eyelid shut Comments: Bilateral: Lyme disease, infectious mononucleosis

Question: What are the three types of botulism infection?

Answer: Foodborne botulism, wound botulism, and infant botulism. Rapid Review Botulism: Patient will be an infant With a history of eating honey Complaining of feeble cry, constipation PE will show symmetric descending paralysis ("floppy baby") Most commonly caused by Clostridium botulinum Treatment is IV botulism Ig

Question: How long before there is a response to oral iron therapy in a patient with iron deficiency anemia?

Answer: Four to eight weeks.

Question: What is the HELLP syndrome?

Answer: HELLP syndrome is a severe form of preeclampsia characterized by hemolysis, elevated liver enzymes and low platelets. Rapid Review Preeclampsia Patient > 20 wks with HTN + proteinuria (30 mg/dL or 300 mg/24 hours) Mild: >140/90 Severe: >160/110 + end organ damage Preeclampsia < 20 weeks gestation: molar pregnancy Bed rest, left lateral decubitus position Eclampsia ppx: Mg sulfate BP control: hydralazine/labetalol Severe preeclampsia: admission

Question: What should babies born to HBV infected women receive?

Answer: Hepatitis B immune globulin (HBIG) and hepatitis B vaccine (HBV). Rapid Review Acute Hepatitis HAV: fecal-oral, shellfish, alone (no carrier), asymptomatic, acute HBV: HBsAg: active infection Anti-HBs: recovered or immunized Anti-HBc IgM: early marker of infection, positive in window period Anti-HBc IgG: best marker for prior HBV HBeAg: high infectivity Anti-HBeAb: low infectivity HCV: IVDA, chronic, cirrhosis, carcinoma, carrier HDV: dependent on HBV coinfection HEV: fecal-oral (enteric) high mortality rate among pregnant (expectant) patients, epidemics, HAV and HEV are fecal-oral: "The vowels hit your bowels" Autoimmune hepatitis: young females Alcoholic hepatitis: moderate transaminase elevation, AST>ALT Supportive rx

Question: What is a complication of tension headache?

Answer: Rebound headaches with frequent analgesic use. Rapid Review Tension Headache Patient will be complaining of bilateral, non-pulsating, bandlike pain PE will show neck muscle tenderness Most commonly caused by stress Treatment is NSAIDs Comments: most common type of headache

Question: What is rhinitis medicamentosa?

Answer: Rebound vasodilation and nasal obstruction resulting from prolonged use of topical nasal decongestants. Rapid Review Sinusitis MCC: viral URI Pain over sinus Purulent rhinorrhea ABX for refractory cases

Question: When should a patient with prostatitis be referred to a urologist?

Answer: Toxic patients, complicated infections, chronic cases and those that don't resolve with standard therapy. Rapid Review Prostatitis Patients <35: N. gonorrhea, C. trachomatis Patients >35: E. Coli Fever, chills, perineal/back pain, dysuria Ceftriaxone or ofloxacin and doxycycline x 30 days (patients <35) Ciprofloxacin or TMP/SMX x 30 days (patients >35)

Question: Prior to electrical cardioversion in a hemodynamically stable patient with atrial fibrillation that has been present for > 48 hrs, what test needs to be done to assess for atrial clot formation?

Answer: Transesophageal echocardiogram. Rapid Review Atrial Fibrillation Alcohol Irregularly irregular No P waves Narrow QRS unless conduction block or accessory pathway Unstable: cardioversion Stable: Rate control with CCBs, ßBs < 48 hours duration: cardiovert to sinus rhthym > 48 hours duration: anticoagulate, echo to r/o thrombus, then cardioversion

Question: What is the most common cause of intussusception in adults?

Answer: Unlike children where most intussusception is idiopathic, 65% of adult cases are due to tumors (either benign or malignant). Rapid Review Small Bowel Obstruction Patient with a history of pelvic surgery Complaining of bilious vomiting PE will show high pitched bowel sounds X-ray will show dilated bowel, air fluid levels, "stack of coins" or "string of pearls" sign Diagnosis is made by CT Treatment is NGT, surgery

A 55-year-old obese man presents to the clinic complaining of fatigue and poor concentration for the past six months. His wife reports that he snores nightly and often sounds like he gasping for air. Which of the following would most likely be found on physical exam? A large neck circumference Coarsening of facial features Increased skin pigmentation Moon facies

Correct Answer ( A ) Explanation: A large neck circumference would most likely be found on physical exam in this man with symptoms concerning for obstructive sleep apnea. Cardinal features of obstructive sleep apnea include irregular and abnormal respiratory patterns, loud snoring and snorts during sleep. Symptoms attributable to disrupted sleep include sleepiness, fatigue and poor concentration. Well-defined risk factors for obstructive sleep apnea include older age, male gender, obesity, craniofacial abnormalities, and upper airway soft tissue abnormalities. Tonsillar hypertrophy, macroglossia, retrognathia/micrognathia, and upper airway mass lesions can contribute to upper airway narrowing. The physical exam can be normal, although obesity, elevated blood pressure, a narrow airway, and a large neck circumference are common. Obstructive sleep apnea is more strongly correlated with an increased neck size or waist circumference than general obesity and is particularly prominent among men who have a collar size greater than 17 inches and women who have a collar size greater than 16 inches. Moon facies (D) is associated with Cushing syndrome, not obstructive sleep apnea. Increased skin pigmentation (C) is sometimes seen in Addison disease and coarsening of facial features (B) is seen with acromegaly. Neither of these physical exam findings are associated with obstructive sleep apnea.

A 30-month-old boy presents with respiratory distress. He was diagnosed with croup one week ago and received a single dose of dexamethasone. His cough had nearly resolved until yesterday, when coughing worsened, and he developed nasal congestion. This evening, he developed inspiratory stridor, increased work of breathing, and a fever to 103°F. What is the most likely diagnosis? Bacterial tracheitis Bronchiolitis Foreign body aspiration Recurrent croup

Correct Answer ( A ) Explanation: Bacterial tracheitis is a rare upper airway obstruction in children. It may occur in isolation or following a viral respiratory illness, including croup. Bacterial tracheitis should be suspected in any child with clinical worsening after improvement of croup symptoms. In children without a preceding illness, symptom onset may begin insidiously with one to three days of viral upper respiratory infections prior to decompensation with respiratory distress, inspiratory stridor, and high fevers. Other children will not demonstrate prodromal symptoms and will present acutely toxic and in extremis. The most common causative organism is Staphylococcus aureus. Less common causes include S.pneumoniae, S.pyogenes, M.catarrhalis, and H.influenzae. Endoscopy is necessary to establishing a definitive diagnosis, but it should take place in a setting where airway supplies and personnel are readily available, such as the operating room or the intensive care unit. Bronchiolitis (B) would be expected to show signs of lower respiratory tract involvement such as crackles and wheezes on examination. Moreover, the child's acute worsening of symptoms of upper respiratory infection following an episode of croup should raise suspicion for bacterial superinfection rather than acquisition of a new viral respiratory infection. Foreign body aspiration (C) may present with signs of upper airway obstruction such as inspiratory stridor. In contrast to bacterial tracheitis, fever is typically absent in foreign body aspiration, and the onset of symptoms is very abrupt. Additionally, in the above child, a recent episode of croup should raise suspicion for a bacterial superinfection. Recurrent croup (D) would be unexpected in an otherwise healthy child who had been improving throughout the week following treatment with dexamethasone.

A 30-year-old-man presents to office with headaches that have been occurring every day, lasting 3-4 hours associated with severe throbbing unilateral retroorbital pain and ipsilateral lacrimation and nasal congestion. This has been occurring for months and the patient complains of agitation and restlessness when he is having a headache. Which of the following is the most likely diagnosis? Cluster headache Migraine headache Temporal arteritis Tension headache

Correct Answer ( A ) Explanation: Cluster headaches occur most often in middle-aged men. They are described as recurrent, unilateral, and last from 15 minutes to 3 hours. The headache is nonpulsatile and constant, frequently occurring at night. Clinical findings include ipsilateral conjunctival injection, lacrimation, and nasal discharge. Cluster headaches classically take place in groups over days to weeks, occurring at the same time of day and in the same location. The neurologic exam should be normal. Treatment of an acute attack includes 100% oxygen, triptans, ergotamines, intranasal lidocaine, and butorphanol. Prophylaxis includes avoiding triggers such as alcohol, stress, and medications causing vasodilation. To prevent acute recurrence, a prednisone burst and taper may be helpful. Migraine headaches (B) are more common in women and 90% of migraine sufferers have a positive family history. Temporal arteritis (C) occurs most commonly in older woman, with reproducible temporal pain without the symptoms as described above. Tension-type headaches (D) can be episodic or chronic. Headaches last for 30 minutes to 7 days, with at least two of the following pain characteristics: pressing or tightening (nonpulsating) quality, mild or moderate intensity, bilateral location, no aggravation by walking, climbing stairs, or similar routine physical activity.

A 4-year-old boy presents to the office with 7 days of fever. On HEENT exam you note bilateral conjunctivitis, some fissuring of his lips and palpate a cervical lymph node. The cardiovascular and pulmonary exams are normal. Skin exam reveals a macular rash on his chest and skin desquamation on his feet. Which of the following is the most likely diagnosis? Kawasaki disease Lyme disease Measles Scarlet fever

Correct Answer ( A ) Explanation: Kawasaki disease typically presents in children younger than 4 years, most commonly at age 2. It is more common in males and occurs more frequently in the winter and spring months. The diagnosis requires the criteria found in the image below. Laboratory findings may reveal thrombocytosis, leukocytosis, and a normocytic anemia. Acute phase reactants (CRP, ESR, and alpha-1 antitrypsin) are often elevated. Antinuclear antibodies (ANA) and rheumatoid factor (RF) are not elevated in Kawasaki disease. The main complication is coronary artery disease. Treatment includes high dose aspirin (80 mg/kg/day) and IVIG. Aspirin maintenance (5 mg/kg/day) is continued for 2 months after high dose spirin is administered for two days. A 2D-echcardiogram should also be performed at the time of diagnosis, 2-3 weeks and 6-8 weeks after the diagnosis is made. Lyme disease (B) is characterized by fever, arthraligias, carditis, and neurologic manifestations. The characteristic rash, erythema migrans, takes on a "bull's eye" appearance. Measles (C) is also associated with an exudative conjunctivitis but also includes cough, coryza, Koplik spots, cephalocaudal development of the typical rash, and leukopenia. Ocular findings are rare in Scarlet fever (D).

An elderly man presents with 5 months of bruising, back pain, progressive leg weakness and distal leg paresthesias. Examination reveals flaccid weakness and areflexia in the legs. Laboratory testing reveals hypercalcemia, thrombocytopenia, anemia and elevated serum creatinine. A lumbar radiograph shows an L4 vertebral body fracture. Which of the following is the most likely diagnosis? Multiple myeloma Myelomeningocele Myelopathy Myositis ossificans

Correct Answer ( A ) Explanation: Multiple myeloma is a malignant proliferation of plasma cells which produces excessive monoclonal paraproteins. Symptoms include bone pain, pathologic fractures, recurrent infections, spinal cord compression, malaise, weakness, anemia, bleeding, renal failure and hypercalcemia. Patients may be asymptomatic, and diagnosis may be incidental. Presentation often includes bone pain (commonly associated with atraumatic pathologic axial spine fractures) and abnormal laboratory results (especially abnormal serum/urine protein electrophoresis, anemia and hypercalcemia). Associated findings include macular detachment, retinal hemorrhage or cotton-wool spots, ecchymoses and pallor, cardiomegaly, hepatosplenomegaly and carpal tunnel syndrome. Bone and PET scanning, as well as plain radiography and MRI, are routinely used in the diagnostic work-up. Myelomeningocele (B), a form of spina bifida, is a congenital disorder. This neural tube defect is marked by failure of closure of the posterior spinal elements. It results in open protrusion of the meninges and spinal cord. Myelopathy (C) is a general term for spinal cord injury. It has several etiologies, of which, cervical spine injury is quite common. Myelopathy is the constellation of upper motor neuron signs (spastic weakness, spasticity, hyperreflexia, positive Babinski and Hoffman's signs), paresthesias and numbness and genitourinary dysfunction. Myositis ossificans (D), also known as heterotopic ossification, involves soft tissue calcification, typically about a joint which has suffered trauma, immobilization or surgery. The quadriceps and gluteals are commonly affected sites.

A 16-year-old man presents to the ED complaining of 3 days of nasal rhinorrhea, cough, myalgias, and generalized malaise. After a coughing episode yesterday, he developed pleuritic chest pain radiating to the left neck. His vital signs are BP 130/70 mm Hg, HR 76, RR 16, T 36.6°C and pulse oximetry 98% on room air. You obtain the chest radiograph seen above. Which of the following is the most appropriate next step in management? Discharge home with close follow-up Endoscopy Needle decompression Tube thoracostomy

Correct Answer ( A ) Explanation: Spontaneous pneumomediastinum is the presence of air or gas in the mediastinum. The source of air usually originates from the respiratory or gastrointestinal tract. Spontaneous pneumomediastinum usually results from rupture of alveoli, particularly after a strenuous Valsalva maneuver, leading to air dissecting along peribronchovascular sheaths and spreading into the mediastinum. The air then moves through mediastinal fascia planes and spreads to subcutaneous tissues of the thorax, upper limbs, and cervical region. Patients commonly report transient stabbing chest pain that may radiate to the shoulders, arms, or back. The finding of Hamman's sign (crunching sound heard on auscultation of the mediastinum with each heartbeat) is suggestive of pneumomediastinum but has a low sensitivity. Spontaneous pneumomediastinum is generally a benign, self-limited condition that usually does not require hospital admission. Secondary causes of pneumomediastinum such as Boerhaave's syndrome are more serious, and treatment is aimed at the underlying disorder. Endoscopy (B) is not necessary for spontaneous pneumomediastinum but is often needed to evaluate esophageal causes. Needle decompression (C) and tube thoracostomy (D) are treatments for pneumothorax, not pneumomediastinum.

A five-year-old boy is brought by his parents to the clinic for a limp. He has been having fever and bone pain for two weeks. For the past few days, the parents noted that he was having difficulty walking. Physical exam reveals multiple inguinal nodes that are nontender, firm, rubbery, and 20 mm in diameter. There is no evidence of point tenderness on palpation. Laboratory testing shows anemia, thrombocytopenia, and neutropenia. There are lymphoblasts on peripheral smear. Which of the following is the most likely diagnosis? Acute lymphocytic leukemia Epstein-Barr virus infection Immune thrombocytopenia Osteomyelitis

Correct Answer ( A ) Explanation: The boy presents with limping, bone pain, fever, lymphadenopathy and lymphoblasts on peripheral smear. These findings are suspicious for acute lymphocytic leukemia (ALL), also known as acute lymphoblastic leukemia. Acute leukemia comprises approximately 30 percent of all childhood malignancies. Each year in the United States, approximately 2500 to 3500 new cases of ALL are diagnosed in children. The peak incidence occurs between two and five years of age. Also, it occurs more commonly among boys than girls. The most common presenting symptoms of ALL are nonspecific, which may include fever, bleeding, bone pain, lymphadenopathy. Unexplained persistence of any of these common signs or symptoms should prompt consideration of malignancy as a possible cause. Most children with ALL have anemia, thrombocytopenia, and lymphoblasts on peripheral smear. Approximately 50 percent of children have WBC counts < 10,000/microL, and 20 percent have an initial leukocyte count > 50,000/microL. Approximately one-half of children with ALL present with bleeding, and three-quarters have a platelet count < 100,000/microL at the time of diagnosis. Epstein-Barr virus infection (B) causes infectious mononucleosis that begins with malaise, headache, and low-grade fever before development of tonsillitis and pharyngitis, cervical lymph node enlargement and tenderness. Immune thrombocytopenia (C) presents with the sudden appearance of a petechial rash, bruising, and bleeding in an otherwise healthy child. Osteomyelitis (D) usually presents acutely with fever, constitutional symptoms, focal findings of bone inflammation (warmth, swelling, and point tenderness), and limitation of function (limp and limited use of extremity).

A five-year-old girl is brought by her father to the clinic because of a cat bite. Yesterday, she was playing with a neighbor's cat that suddenly bit the girl's left hand. Her wound was immediately cleaned. The following day, the father noted increased swelling and redness of the girl's left hand. On physical examination, she has normal vital signs, and on the left hand are two puncture wounds with a surrounding 1 cm diameter of erythema and swelling. Which of the following is the best treatment for a suspected Pasteurella sp. soft tissue infection? Amoxicillin-clavulanate Clindamycin Erythromycin Vancomycin

Correct Answer ( A ) Explanation: The girl has a soft tissue infection from a cat bite. Clinical soft tissue infections with Pasteurella multocida usually occur after cat bites, cat scratches, or dog bites but may also occur following cat or dog licks of non-intact skin. P. multocida wound infections characteristically have a very rapid development of an intense inflammatory response. Most patients develop symptoms within 24 hours of the initial injury, and as early as three hours after a cat bite. Pain and swelling are prominent. Purulent drainage is noted in about 40 pecent of patients, lymphangitis in about 20 percent, and regional adenopathy in 10 percent. Cellulitis often occurs within 24 to 48 hours. Necrotizing fasciitis may occur. Pasteurella is the first organism to consider in any patient who presents with a soft tissue infection following cat scratches or cat or dog bites or licks. The diagnosis of P. multocida infection is made by isolation of the organism in culture. Pasteurella sp., including P. multocida, are usually susceptible to a number of antibiotics, including amoxicillin-clavulanate, piperacillin-tazobactam, doxycycline, fluoroquinolones, advanced cephalosporins, and carbapenems. Pasteurella multocida are usually resistant to clindamycin (B), erythromycin (C), and vancomycin (D) and are not the best therapy for this infection.

A 42-year-old man complains of difficulty washing his face and combing his hair with his right hand. On examination a nodule, band, and slight contracture are noted in the palm proximal to the fourth finger. This patient's symptoms are associated with which one of the following? Diabetes mellitus Hyperparathyroidism Hyperthyroidism Hypothyroidism

Correct Answer ( A ) Explanation: The patient has Dupuytren's disease (Dupuytren's contracture), which is most common in men over 40 years of age. It is a progressive condition that causes the fibrous fascia of the palmar surface to shorten and thicken. Patients present with one or more small, pitted nodules on the palm, which slowly progress to contracture of the fingers. Patients usually have difficulty with tasks such as face washing, hair combing, and putting their hands in their pockets. Grade 1 disease initially can be managed expectantly, but injecting the nodule with a steroid injection can be helpful. Surgery is recommended if function is impaired, contracture is progressing, or severe deformity is disabling. There is a strong association between diabetes and Dupuytren's disease. Studies have found a 3 to 33 percent prevalence of Dupuytren's in patients with diabetes; however, these patients tend to have a mild form of Dupuytren's with slow progression. Greater alcohol intake per week is associated with increased risk of Dupuytren's disease, but most patients with the disease are not alcoholics. Smoking also increases the risk of the disease. Although the reason for this association is unclear, microvascular changes in smokers may play a role. Hyperparathyroidism (B) is a disease caused by a disregulation in calcium. If symptoms are present, they are attributable to hypercalcemia and may include weakness, easy fatigability, anorexia, or anxiety. Untreated hypothyroidism (D) can contribute to hypertension, dyslipidemia, infertility, cognitive impairment, and neuromuscular dysfunction. Hypothyroidism does not have any correlation with dupuytren's disease. Hyperthyroidism (C) causes anxiety, hyperactivity, heat intolerance, muscle weakness, hand tremors and much more. However it does not cause dupuytren's disease.

A 35-year-old man presents with headache, nausea, and lightheadedness. The headache started 1 day after a flight from Boston to Denver. He describes the headache as bifrontal and throbbing. The patient has normal vital signs and his neurologic exam is normal. The patient's presentation is consistent with which of the following disorders? Acute mountain sickness High-altitude cerebral edema Migraine headache Subarachnoid hemorrhage

Correct Answer ( A ) Explanation: The patient has acute mountain sickness (AMS). In order to diagnose AMS, the patient must have a recent gain in altitude which occurred at least several hours ago, a headache, and one of the following additional symptoms: gastrointestinal upset, generalized weakness, fatigue, dizziness, lightheadedness, or insomnia. The headache is generally bitemporal and throbbing. Headaches are generally worse at night and when moving to an upright position. The pathophysiology of AMS is not fully understood, but is thought to be caused by hypoxia which leads to intracranial vasodilation and cerebral edema. Insomnia is caused by nocturnal, periodic breathing. The cornerstone of management of AMS is to halt ascent until symptoms improve. Headaches can be treated with acetaminophen or ibuprofen. Supplemental oxygen may also reduce symptoms. Acetazolamide can be used to stimulate breathing and can be helpful for insomnia. It is a carbonic anhydrase inhibitor which causes a renal bicarbonate diuresis. This causes a metabolic acidosis which stimulates increased ventilation and thereby improves arterial oxygenation. If symptoms do not improve or worsen with conservative management, the patient should descend to lower altitudes. High-altitude cerebral edema (B) is the most severe form of AMS and manifests when cerebral edema becomes profound enough to cause global cerebral dysfunction. Signs of high-altitude cerebral edema (HACE) include ataxia, confusion, and altered mental status. It can progress to brain-stem herniation and death. Management of HACE includes immediate descent, supplemental oxygen, acetazolamide, and steroids. Portable hyperbaric chambers can be used as a temporizing measure if descent cannot be immediately achieved. A typical, migraine headache (C) would be characterized by a unilateral, throbbing headache. Patients usually have a history of similar headaches. In the setting of recent ascent to high altitude, as in this patient, AMS is more likely. A subarachnoid hemorrhage (D) usually presents with sudden onset, severe headache.

A 32-year-old man presents with acute onset right-sided facial weakness that he first noticed upon waking this morning. You ask him to smile as seen in the image above. Which of the following clinical features, if present, suggests a poorer prognosis? A vesicle found on the right tympanic membrane Dysgeusia Hyperacusis Unilateral symptoms

Correct Answer ( A ) Explanation: The patient has idiopathic facial nerve paralysis (Bell's palsy). Bell's palsy is characterized by rapid onset of unilateral facial paralysis. More than half of patients have a prodromal viral illness (rhinorrhea, myalgias, etc). Bell's palsy is associated with ear pain, decreased tearing, an overflow of tears onto the cheek (epiphora), hyperacute hearing (hyperacusis), and dysgeusia (an impairment of taste). Idiopathic facial nerve paralysis involves the lower motor neuron. Features that suggest an upper motor neuron cause include a slow progression of symptoms, recurrent symptoms, and intact forehead movement. A herpetiform vesicular eruption on the tympanic membrane, pinna, external auditory canal, soft palate, oral cavity, face, or neck suggests a diagnosis of Ramsay Hunt syndrome (herpes zoster oticus). In Ramsay Hunt syndrome, the pain is more severe than in Bell's palsy, and the prognosis is worse, with a lower incidence of complete facial recovery and the possibility of associated sensorineural hearing loss. Dysgeusia (B), hyperacusis (C), and unilateral symptoms (D) are not associated with a poorer outcome. Bilateral facial nerve paralysis suggests a diagnosis of Lyme disease, which is treated differently than Bell's palsy or Ramsay Hunt syndrome. Bell's palsy and Ramsay Hunt syndrome are treated with steroids and antivirals for 7-10 days.

A 17-year-old G1P0 woman at 25-weeks gestation presents with intermittent blurred vision. On presentation, she is asymptomatic. Vital signs are HR 84, BP 175/113, oxygen saturation 97%. Physical examination reveals 2+ pitting edema on both lower extremities and urinalysis has 3+ protein on dip. Which of the following is most likely indicated? Admit for further obstetrics evaluation Antiepileptic medications Arrange follow up with the patient's obstetrician Emergency cesarean section

Correct Answer ( A ) Explanation: This patient presents with severe preeclampsia and should be admitted for further obstetric evaluation. Preeclampsia is defined as gestational hypertension (>140/90 mm Hg) with proteinuria (>300 mg/24 hr) that occurs after 20-weeks gestation. Progression from preeclampsia to eclampsia (hypertension, proteinuria and seizures) is unpredictable and can occur rapidly. In preeclampsia, patients may be asymptomatic. In severe disease, typically defined as a blood pressure >160/110 mm Hg, patients may have associated epigastric or liver tenderness, visual disturbances or severe headaches. These patients should be admitted for further management. Treatment for these patients is the same as in eclampsia. The goal of treatment is prevention of seizures or permanent maternal organ damage. Magnesium should be given for seizure prophylaxis. Antiepileptic medications (B) are not indicated for seizure prophylaxis. Patients with mild preeclampsia without symptoms or isolated maternal hypertension can be managed as outpatients (C) but not those with severe preeclampsia. Ultimately, delivery is the potentially curative therapy for preeclampsia but at this stage in pregnancy, an emergent cesarean section (D) would not be indicated. [Of note: Recent ACOG guidelines state that proteinuria is not required to make a diagnosis of preeclampsia if there is evidence of end-organ damage as noted above.Massive proteinuria is also not necessary for a diagnosis of severe preeclampsia.]

A 32-year-old woman presents six hours after ingesting 40 tablets of regular-strength (325 mg) aspirin in a suicide attempt. She is lethargic with a heart rate of 106 beats/minute, blood pressure of 142/84 mm Hg, respiratory rate of 30 breaths/minute, and a temperature 38.5 0C. What abnormality would be expected on her arterial blood gas? Mixed respiratory acidosis with a metabolic alkalosis Mixed respiratory alkalosis with a metabolic acidosis Pure metabolic acidosis Pure respiratory alkalosis

Correct Answer ( B ) Explanation: Aspirin toxicity results in a complex acid-base disturbance. Salicylates stimulate the respiratory center resulting in tachypnea, with a subsequent decrease in pCO2 and respiratory alkalosis. Cell metabolism is also interrupted, leading to the production of lactic acid and ketoacids and an elevated anion gap metabolic acidosis. The mixed respiratory alkalosis with the elevated anion gap metabolic acidosis can result in a near-normal pH. In the late stages of toxicity, as the patient becomes progressively more fatigued with associated electrolyte abnormalities and dehydration, a respiratory acidosis can occur which signals impending cardiovascular collapse. A mixed respiratory acidosis with a metabolic alkalosis (A) is not seen in salicylate toxicity. A pure metabolic acidosis (C) is unusual in adults, but can be seen in young children in the early stages of toxicity. A pure respiratory alkalosis (D) is also not typical of salicylate toxicity given its effect on cellular metabolism and subsequent lactic acid and ketoacid production.

A 30-year-old woman is pregnant with her third child. Her medical history is significant for cocaine abuse. She delivered her first and second low-birth-weight children preterm. Both of these children have congenital abnormalities. Which of the following diseases is this third fetus most at risk of developing? Cerebellar ataxia Cerebral palsy Charcot-Marie-Tooth disease Cystic fibrosis

Correct Answer ( B ) Explanation: Cerebral palsy (CP) is a neurodevelopmental disease caused by a non-progressive CNS insult that occurred during fetal or infant development. It is characterized by movement and posture abnormalities which are associated with a seizure disorder or disturbances of sensation, cognition, perception, behavior and communication, or both. Risk factors include birth weights <2,000 grams, sibling history of congenital malformations, maternal mental retardation or substance abuse (cocaine causes intrauterine ischemia) and birth injury. Three main classes of CP exist: (1) Dyskinetic (extrapyramidal) CP, in which intrapartum asphyxia leads to a need for postnatal resuscitation and eventual tetraplegia, pseudobulbar palsy, choreathetosis and dystonia; (2) Spastic (pyramidal) CP, which occurs gradually between the ages of 4-6 months, and manifests as extremity paralysis (spastic diplegia, triplegia or tetraplegia), gait abnormalities and decreased intelligence; (3) Mixed CP, in which both types of symptoms are present. The major complications which carry over into adulthood are mental retardation, learning disabilities, seizure, musculoskeletal pain, osteoarthritis, hip and back problems, bowel and bladder problems and gastroesophageal reflux. Treatment is based on aggressive and lifelong physical, occupation and speech rehabilitation. Cerebellar ataxia (A) is a childhood, not fetal, disease caused by previous infection with varicella-zoster virus, Epstein-Barr virus and echovirus. Charcot-Marie-Tooth disease (C), the most common inherited neurologic disorder, is characterized by myelin and axonal abnormalities that cause distal > proximal weakness and atrophy. Maternal age and substance abuse history are not specific risk factors for this condition. The biggest risk factor for cystic fibrosis (D) is family history, not maternal substance abuse.

Which of the following is true regarding the management of a subungual hematoma? Antibiotics should be prescribed to patients after trephination of uncomplicated subungual hematomas Complete nail removal may be necessary if the nail bed has been significantly damaged Trephination is contraindicated when the subungual hematoma is associated with a distal phalanx fracture Trephination is indicated only when the hematoma involves greater than 50% of the nail bed

Correct Answer ( B ) Explanation: Complete nail removal may be necessary if the damage to the nail bed is significant. A subungual hematoma is the result of a laceration to the nail bed. If the nail is stable and firmly adherent to the nail bed and the surrounding tissues are not significantly damaged, there is no reason to remove the nail and repair the underlying nailbed laceration. The nail itself will keep the nail bed laceration well approximated. Nail trephination in this scenario often results in a good cosmetic endpoint. Nail trephination is preformed under sterile technique with adherence to universal precautions. Local anesthesia is generally unnecessary for uncomplicated hematoma trephination. More complex injuries, including those in which nail removal is necessary, will require a digital block for patient comfort. Most commonly hot cautery (with a heated paperclip or disposable cautery device) is used. The device is inserted perpendicular to the nail in the center of the hematoma. A pop will be felt as the device passes through the nail and at this point the device should be removed to avoid damage to the nail bed. Blood will come out of the hole and the nail will return to a normal color. Gentle downward pressure applied by the patient will help blood escape. Occasionally large hematomas require more than one trephination site for adequate drainage. After the procedure, the area should be cleaned and dressed with a dry dressing. Splints are applied as indicated. The patient is instructed to keep the digit dry for 2 days. Alternative trephination techniques include the use of an 18-gauge needle twisted into the nail. Also a small 29-gauge needle can be inserted parallel and under the nail, past the hematoma site. This is useful for hard to drain second-fifth toe subungual hematomas. Subungual hematoma is often the result of slamming a finger in a door, striking a finger with an object or dropping an object onto a toe. This can result in a distal phalanx fracture as well. This is not a contraindication to nail trephination (C). The general rule is that all painful, acute (24-48 hours old) subungual hematomas should be drained irrespective of the size (D). Some sources cite that only hematomas greater than 50% should be drained, however smaller hematomas should be drained if the patient is experiencing significant discomfort. There is little risk of infection after trephination of an uncomplicated subungual hematoma therefore antibiotics are not prescribed (A).

A 33-year-old woman with a history of Celiac disease presents with a chronic, pruritic papulovesicular lesions occurring symmetrically over the extensor surfaces of her elbows and knees. Several vesicles have a crusted appearance. Which of the following is the most likely diagnosis? Atopic dermatitis Dermatitis herpetiformis Erythema nodosum Pyoderma gangrenosum

Correct Answer ( B ) Explanation: Dermatitis herpetiformis an intensely pruritic papulovesicular rash, is pathognomonic for Celiac disease. It most commonly occurs at the extensor surfaces of the elbows and knees as well as buttocks, and may occur even in patients who appear to have no gastrointestinal problems. Diagnosis of dermatitis herpetiformis is made by visualizing granular IgA deposition on immunofluorescence of a skin biopsy. As with the gastrointestinal manifestations of Celiac disease, dermatitis herpetiformis typically resolves with full elimination of gluten from the diet. Atopic dermatitis (A) in adults usually presents with dry, pruritic areas of inflammation at the elbow and knees flexor surfaces. It is associated with underlying allergic conditions like asthma and allergic rhinitis. Erythema nodosum (C) is characterized by painful, red nodules commonly in a pretibial distribution, which fade into bruises after approximately 2 weeks. It is seen in with systemic illnesses including sarcoidosis and inflammatory bowel diseases as well as after using certain antibiotics and contraceptives. Pyoderma gangrenosum (D) typically presents as an inflamed papule or pustule that progresses to a painful ulcer with a violaceous, purulent base; it is associated with underlying autoimmune conditions including inflammatory bowel disease.

A patient with alcohol abuse and cirrhosis presents with acute upper gastrointestinal bleeding, hypotension and tachycardia. His past medical history also includes portal hypertension. You suspect ruptured esophageal varices as the source of bleeding. In addition to cardiopulmonary stabilization and blood transfusion, which of the following is the most appropriate emergent interventional treatment? Distal splenorenal shunt Endoscopic band ligation Large volume paracentesis Transjugular intrahepatic portosystemic shunt

Correct Answer ( B ) Explanation: Esophageal varices are dilated submucosal veins which commonly occur in the lower third of the esophagus. They are mainly due to portal hypertension secondary to cirrhosis. They typically present with massive upper gastrointestinal bleeding. Emergency care is based on hemorrhage control, maintaining cardiopulmonary function and correcting concurrent cirrhosis-induced coagulopathies. Blood transfusions are often necessary. Therapeutic endoscopy, with variceal ligation, banding or sclerotherapy, is considered by many to be the definitive emergent treatment. Terlipressin, a vasopressin analogue, and octreotide, a somatostatin analogue, can also be used in the emergent care of bleeding esophageal varices. Refractory cases may require balloon tamponade. Transjugular intrahepatic portosystemic shunt (D) and distal splenorenal shunt (A) are two procedures used for treating underlying portal hypertension. They are not used in the emergent treatment of variceal bleeding, but reserved to treat such patients once they are stabilized. Large volume paracentesis (C) is used to treat refractory ascites, which is another complication of cirrhosis. It is not used to control bleeding varices.

A previously healthy 32-year-old man presents to your office with a complaint of tingling in both feet. He says that the tingling sensation started 3 days ago and now his lower legs feel weak and he is having difficulty walking. In addition, he is having trouble chewing his food. Physical exam shows absent patellar and Achilles reflexes. Which of the following is the most likely diagnosis? Diabetic neuropathy Guillain-Barré syndrome Multiple sclerosis Myasthenia Gravis

Correct Answer ( B ) Explanation: Guillain-Barré syndrome is an autoimmune disorder that affects the peripheral nervous system. The classic presentation involves symmetric, progressive muscle weakness along with weak or absent deep tendon reflexes. Facial weakness occurs in more than 50% of individuals. Diagnosis is generally based on clinical signs and symptoms, but may be confirmed by cerebrospinal fluid analysis which will reveal elevated protein with a normal white blood cell count. Treatment includes supportive care, admission to the ICU for monitoring of respiratory and cardiac function, plasma exchange and IVIG. Diabetic neuropathy (A) is one of the most common complications of diabetes and affects the peripheral nervous system in a symmetric, distal to proximal pattern. However, individuals do not lose their deep tendon reflexes. Multiple sclerosis (C) is an autoimmune inflammatory disease involving the central nervous system. Women are more commonly affected than men. Symptoms include vision changes and transient, intermittent muscle weakness. Myasthenia gravis (D) is an autoimmune disorder in which acetylcholine receptors are blocked by antibodies. It is classically described as presenting with fluctuating muscle weakness, ptosis and diplopia. Symptoms are usually worse later in the day.

A 4-year-old is brought to the emergency department by his parents after they found him in the backyard shed choking and gagging. Before this event, he was otherwise healthy. His temperature is 101°F, heart rate is 95, blood pressure is 100/60, and respiratory rate 40. On exam, he appears sleepy but continues to cough. Pulmonary findings include moderate retractions and diffuse wheezes. A chest X-ray shows patchy infiltrates. What is the most likely diagnosis? Foreign body aspiration Hydrocarbon ingestion Organophosphate ingestion Status asthmaticus

Correct Answer ( B ) Explanation: Hydrocarbons (e.g. lamp oil, gasoline, lighter fluid, turpentine, benzene, kerosene) can be aspirated easily when ingested and may cause pneumonitis with volumes as low as < 1 mL. The lower viscosity compounds distribute across a larger lung surface. Symptoms of aspiration are non-specific and include grunting, gagging, choking, tachypnea, fever, retractions and persistent coughing. These findings may be delayed, so asymptomatic children should be observed for several hours. The chest X-ray typically shows diffuse bilateral infiltrates. Airway support is important because symptoms can evolve into respiratory failure. Hydrocarbons cause acute respiratory distress syndrome (ARDS) by inactivation of type II pneumocytes, which leads to surfactant deficiency. Foreign body aspiration (A) can cause similar respiratory symptoms, but the chest X-ray will either be normal or show a foreign body if it is radio-opaque. Organophosphate ingestion (C) can cause bronchospasm, but symptoms are not limited to the respiratory tract and include diarrhea, urination, miosis, emesis, lacrimation and salivation. An individual with status asthmaticus can have similar respiratory symptoms, but the fever and chest X-ray findings are inconsistent with the disease process. This patient has no history of wheezing, and status asthmaticus (D) generally has a more insidious onset. During an asthma exacerbation, the chest X-ray may show hyperinflation of the lungs, not patchy infiltrates.

Which of the following profiles is consistent with immunity to hepatitis B virus secondary to vaccination? Hepatitis B core antibody positive, Hepatitis B surface antibody positive Hepatitis B surface antigen negative, Hepatitis B surface antibody positive Hepatitis B surface antigen positive, Hepatitis B e antigen positive Hepatitis B surface antigen positive, Hepatitis B surface antibody negative

Correct Answer ( B ) Explanation: In individuals that are vaccinated and immune due to previous vaccination hepatitis B surface antigen will be negative and hepatitis B surface antibody will be positive. This suggests that antibodies have been formed because of vaccination and not exposure to virus. Hepatitis B vaccine is part of routine immunizations in the United States, and as a result, the incidence of HBV has significantly declined. A Cochrane review confirmed that hepatitis B vaccination decreased HBV infection in health care professionals (relative risk = 0.51; 95% confidence interval, 0.35 to 0.73). Because there is a high risk of acquiring HBV from a needle stick injury, health care professionals exposed to HBsAg-positive blood should be given hepatitis B immune globulin after the exposure and started on the hepatitis B vaccine series if not previously vaccinated. The diagnosis of HBV infection requires the evaluation of the patient's blood for hepatitis B surface antigen (HBsAg), hepatitis B surface antibody (HBsAb), and hepatitis B core antibody (HBcAb). Although the presence of HBsAg indicates that the person is infectious, the presence of HBsAb indicates recovery and immunity from HBV infection or successful immunization against HBV. HBcAb appears at the onset of acute HBV infection, but may also indicate chronic HBV infection. Individuals infected with hepatitis B virus are either hepatitis B core IgG positive (A), hepatitis B surface antibody positive, suggesting they have been exposed to the virus (thus creating core antibodies) but may have cleared the infection . Hepatitis B surface antigen positive, hepatitis B e antigen positive (C) suggests chronic ongoing infection, as does hepatitis B surface antigen positive, hepatitis B surface antibody negative (D).

A four-month-old baby presents to your office with symptoms of worsening constipation, poor feeding, listlessness, and generalized weakness for two weeks. Physical exam findings include temperature of 98.6°F, ptosis, poor head control, and poor ability to suck. Which of the following is the most appropriate initial therapy? Bisacodyl Human-derived botulism immune globulin Magnesium citrate Senna

Correct Answer ( B ) Explanation: Infant botulism, while rare, is the most common form of botulism seen in the United States. Raw honey is a frequently mentioned etiology of infant botulism. Other causes include corn syrup and soil and vacuum dust. The initial clinical manifestation is constipation, followed by motor function symptoms such as ptosis, facial and generalized weakness. Clinical presentation and electromyography findings consistent with infant botulism allow for a presumptive diagnosis while confirmatory stool studies are pending. Treatment in infants younger than age one is with human-derived botulism immune globulin (BabyBIG) and should be administered as early as possible in the course of the infection. Equine-derived heptavalent botulinum antitoxin is used in non-infant cases of botulism. Bisacodyl (A) and senna (D) are stimulant laxatives that are not recommended for children younger than three years old. Cathartics containing magnesium, such as magnesium citrate (C) should not be used in the treatment of infant botulism.

Which of the following statements is true regarding the local anesthetic lidocaine? It is a member of the class of ester anesthetics Symptoms of overdose include seizures and cardiac dysrhythmia The maximum dose is 3-5 mg/kg when prepared with epinephrine The mechanism of action is via calcium channel blockade

Correct Answer ( B ) Explanation: Lidocaine overdose results in CNS and cardiovascular problems and are secondary to sodium channel blockade. CNS symptoms include dizziness, headache, tingling, tinnitus, sedation, tremors and seizures. Cardiovascular effects include bradycardia, heart block and dysrhythmias. Lidocaine is an amide (A) anesthetic, not an ester. Other local anesthetics in the amide group include bupivacaine, prilocaine, mepicacaine, and etidocaine. Ester anesthetics include procaine, benzocaine, tetracaine, and cocaine. The mechanism of action of lidocaine is via sodium channel blockade, not calcium channel blockade (D). As a result, it provides local anesthesia by blocking nerve conduction. The maximum dose of Lidocaine is 3-5 mg/kg without epinephrine and 5-7 mg/kg with epinephrine (C).

You examine a 12-month old boy in clinic for possible measles exposure. He stayed at his grandmother's house two days ago where he played with his cousin. The mother is concerned because the boy's cousin developed measles that is confirmed by the laboratory. The mother denies any symptoms on the boy. The boy has not had his measles vaccination. Which of the following is the best therapy for the boy? Measles immune globulin Mumps, measles, rubella vaccination Ribavirin Vitamin A

Correct Answer ( B ) Explanation: The boy has been exposed to a confirmed case of measles and therefore needs postexposure prophylaxis. Measles is a highly contagious viral infection characterized by fever and exanthem. Prevention depends upon vaccination of all susceptible individuals. Maintenance of herd immunity is important even in countries with a low incidence of measles. Postexposure prophylaxis for susceptible individuals exposed to measles consists of vaccination within 72 hours of exposure in the absence of a contraindication. Mumps, measles, rubella vaccination (MMR) vaccination can provide some protection or modify the clinical course of measles if administered within 72 hours of measles exposure. For vaccine-eligible individuals aged greater than or equal to 12 months who have been exposed to measles, MMR vaccine should be administered unless there is a contraindication, including individuals who have only received one previous vaccine dose. Administration of vaccine is preferable to administration of immune globulin since vaccination provides active, long-lasting immunity against measles. Measles immune globulin (A) can prevent or diminish the severity of measles if administered to nonimmune individuals within six days of exposure. However, the administration of live measles vaccine within 72 hours of the exposure is preferable to immune serum globulin administration. Data on the use of ribavirin (C) for measles is limited, and there have been no randomized controlled trials to assess its clinical benefit. Giving vitamin A (D) to children with measles is associated with decreased morbidity and mortality. However, vitamin A is not given as post-exposure prophylaxis.

A 63-year-old man presents with fever, chills, low back and perineal pain. His pain is increased with urination and he has both frequency and incomplete voiding. On exam, you note a tender prostate. Which of the following treatments is most likely indicated? Ceftriaxone and azithromycin X 1 dose Ciprofloxacin for 30 days Ciprofloxacin for 7 days Nitrofurantoin

Correct Answer ( B ) Explanation: This patient presents with prostatitis requiring 4-6 weeks of antibiotic therapy with an agent that covers the most common bacterial pathogens. Bacterial prostatitis is primarily caused mainly by Escherichia coli (80%). Other gram negative organisms involved include Klebsiella, Enterobacter, Pseudomonas, and Proteus. Patients present with dysuria, fever, malaise, chills and perineal and low back pain. Rectal examination often reveals a boggy and very tender prostate. Pain is increased with urination and prostatic inflammation often leads to incomplete voiding and frequency. Treatment requires a prolonged course of antibiotics. Appropriate regimens include ciprofloxacin, ofloxacin or trimethoprim/sulfamethoxazole for 30 days. Fluoroquinolones and third-generation cephalosporins are appropriate for inpatient therapy as well. Ceftriaxone and azithromycin (A) are indicated for the treatment of sexually transmitted infections and cover gonorrhea and Chlamydia. Ciprofloxacin (C) is an appropriate antibiotic for prostatitis but needs to be continued for 30 days. Nitrofurantoin (D) is a first line agent in the treatment of uncomplicated cystitis and is ineffective for bacterial prostatitis.

You evaluate a 10-year-old girl in the ED because of headache. She has been complaining of headache, nausea, and dizziness. She was seen in clinic yesterday and was diagnosed with a viral illness. Her symptoms got worse, and she seems confused. The girl denies rhinorrhea, fever, loss of consciousness, trauma, or burns. Her mother and older brother also complain of headache. You obtain labs that show an elevated carboxyhemoglobin level. Which of the following is the most appropriate therapy? Amyl nitrite Methylene blue Oxygen Supportive care

Correct Answer ( C ) Explanation: Carbon monoxide (CO) is the most common gas involved in pediatric exposures. Smoke inhalation is responsible for most inadvertent cases. CO is a colorless, odorless gas produced during the combustion of any carbon-containing fuel. Potential sources include wood-burning stoves, old furnaces, and automobiles. Early symptoms are nonspecific, including headache, malaise, nausea, and vomiting. At higher exposure levels, patients can develop mental status changes, confusion, ataxia, syncope, tachycardia, and tachypnea. Severe poisoning is manifested by coma, seizures, myocardial ischemia, acidosis, cardiovascular collapse, and potentially death. On exam, patients may have cherry-red skin. Evaluation should include a carboxyhemoglobin level in all symptomatic patients, arterial blood gas and creatine kinase in severely poisoned patients, and an ECG in any patient with cardiac symptoms. Treatment requires the administration of 100 percent oxygen to enhance elimination of CO. Severely poisoned patients might benefit from hyperbaric oxygen. Amyl nitrite (A) is the treatment for cyanide poisoning. Methylene blue (B) is the treatment for methmoglobinemia. The goal of supportive care (D) is to support the vital functions until the patient can eliminate the toxin from the system. Supportive care in this case is not enough since the patient needs to be treated with 100 percent oxygen.

A 36-year-old woman presents to the Emergency Department with progressive lower back pain. She denies history of fever or trauma, but the pain worsened acutely when she bent over to pick up a box earlier today. Which of the following features is most concerning for cauda equina syndrome? Decreased patellar reflex on the right Decreased sensation of the lateral foot Pain radiating down both legs Positive straight leg raise

Correct Answer ( C ) Explanation: Cauda equina syndrome is a neurosurgical emergency that results from sudden compression of multiple lumbar and sacral nerve roots. Often the result of massive central disk herniation, it can also be caused by trauma, malignancy, and epidural abscess or hematoma. Patients present with acute onset of lower back pain with pain, weakness, and numbness affecting multiple levels and both legs. Urinary retention is the most consistent examination finding and has a negative predictive value of 99%. Patients may complain of overflow urinary incontinence. Other findings include saddle anesthesia, decreased rectal tone, and fecal incontinence. Emergent neurosurgical consultation is indicated for operative decompression. A decreased patellar reflex (A) is seen with an L4 radiculopathy. Involvement of S1 will cause decreased sensation of the lateral foot (B). A positive straight leg raise (D) suggests an L5 or S1 radiculopathy.

A 23-year-old nulligravida comes to your office for contraception counseling. She has a seizure disorder that is well controlled on carbamazepine. She is a nonsmoker and has no other medical problems or complaints. She is currently in a relationship and does not want to get pregnant in the next several years. Which one of the following contraceptives is the most appropriate? Combined oral contraceptives Etonogestrel/ethinyl estradiol vaginal ring Levonorgestrel intrauterine device Progestin-only pills

Correct Answer ( C ) Explanation: Certain antiepileptic drugs induce hepatic metabolism of estrogen (carbamazepine, oxcarbazepine, phenobarbital, phenytoin, and topiramate). This can potentially lead to failure of any contraceptive that contains estrogen. Therefore progestin only birth control methods would be beneficial to this patient. The levonorgestrel intrauterine device and copper intrauterine device are acceptable choices even for a nulligravida. The levonorgestrel intrauterine device is a progestin only device that can be used for either 3 years or 5 years depending on if it is the Skyla® or the Mirena® device. The copper intrauterine device does not contain hormones and provides effective contraception for up to 10 years. The single-rod implantable progestin system known as the Nexplanon® would also be an acceptable choice for this patient. This implantable device is inserted into the arm and provides effective contraception for 3 years. Permanent sterilization techniques are not recommended for patients whom are young and may desire pregnancy in the future. Progestin-only pills (D) are most effective in women who are exclusively breastfeeding. They are not as effective in pregnancy prevention in other circumstances. Combined oral contraceptives (A) and the etonogestral/ethinyl estradiol vaginal ring (B) contain estrogen and therefore would be contraindicated in this patient whom is taking antiepileptic drugs.

A 14-year-old boy has been described by others as the "school bully." For the past 16 months he stole money from his parents, got into fights at school almost daily, failed to turn in most of his homework, and destroyed multiple items both at home and school. During parent-principal meetings the young boy denies having remorse for his actions and considers himself just a "normal" 14-year-old. Which of the following disorders is this individual suffering from? Antisocial personality disorder Attention deficit hyperactivity disorder Conduct disorder Obsessive compulsive personality disorder

Correct Answer ( C ) Explanation: Conduct disorder criteria involves symptoms from 3 of 4 categories that include aggression to people and animals, destruction of property, deceitfulness or theft, serious violation of the rules. Symptoms must persist for over 12 months with at least 1 criterion for the past 6 months. They must also be under the age of 18. Attention deficit hyperactivity disorder (ADHD) (B) requires patterns of either inattention or hyperactivity or both that interfere with daily life. In either category they must meet six or more symptoms as listed by the DSM-5 manual. Antisocial personality disorder (A) as defined by the DSM-5 requires the individual to be at least 18 years old and three or more criterion: failure to conform to social norms, deceitfulness, impulsivity, recklessness, disregard for others, or lack of remorse for their actions. Obsessive compulsive personality disorder (D) is characterized by a preoccupation with orderliness, perfectionism, and control. The individual lacks flexibility, and openness for change. They also are unaware of the fact that their actions may cause distress to others. They must meet four or more of the criteria as laid out by the DSM-5 manual.

A one-year-old girl is seen in your clinic for a routine visit. Her mother states that she has been eating well and drinking more than 32 ounces of whole milk a day. You order blood work and find a hemoglobin level of 9 g/dl and a mean cell volume of 74. What is the most likely diagnosis? Beta thalassemia trait Hereditary spherocytosis Iron deficiency anemia Lead poisoning

Correct Answer ( C ) Explanation: Dietary iron deficiency is most common between the ages of one and three years of age. This is because cow's milk is a very poor source of iron. A milk intake greater than 16 ounces a day raises the concern of iron deficiency. Milk is associated with satiety and delayed gastric emptying, which limits the consumption of other iron-containing foods. In these patients, the physical exam may only be significant for mild pallor. Universal screening at one year of age is routinely done to identify and address iron deficiency before leading to severe side effects, such as developmental and mental delay. Initial hemoglobin studies may be normocytic and normochromic and then progress to hypochromic and microcytic as the low iron levels begin to take effect. Iron studies done for verification will show a low iron level and high total iron-binding capacity (TIBC). Treatment of iron deficiency anemia depends on the etiology. Children who have gastrointestinal bleeding should have imaging and possible endoscopy to determine the etiology. In dietary iron deficiency, parents should be counseled to limit whole milk intake to less than 16 ounces a day. Treatment consists of the administration of oral iron sulfate. This is continued until one month after hemoglobin levels normalize. The additional one month allows for the replenishment of the body's iron stores. Beta thalassemia trait (A) results in a mild anemia not associated with dietary intake of iron. No specific treatment is required. Hereditary spherocytosis (B) is an autoimmune condition associated with neonatal jaundice and splenomegaly. Lead poisoning (D) is associated with elevated lead levels and may show basophilic stippling on a blood smear.

A 71-year-old woman presents with 2 days of dizziness and "almost passing out." Her ECG shows episodes of alternating bradycardia and tachycardia with narrow QRS complexes. Which of the following is the most likely diagnosis? Atrial flutter Digitalis toxicity Sick sinus syndrome Ventricular tachycardia

Correct Answer ( C ) Explanation: Sick sinus syndrome occurs as a result of disease of the sinoatrial (SA) node. It is associated with tachycardia-bradycardia syndrome in which the sinus rate varies from fast to slow and back again. ECG shows an irregular rhythm with pauses in sinus activity. Management depends on the presentation. For rhythms that are too slow, the patient may require a pacemaker. Whereas rhythms that are too fast may be treated with calcium channel blockers or beta-blockers. Atrial flutter (A) is associated with regular tachycardia and saw tooth appearance of P waves. Digitalis toxicity (B) is associated with a high degree AV block. Ventricular tachycardia (D) is not associated with a bradycardia.

A 17-year-old boy presents with 2 days of nasal congestion and headache. Examination reveals a well-appearing boy with tenderness to palpation over the left maxillary sinus and nasal congestion and discharge. The patient is afebrile. Which of the following managements is most likely indicated? Amoxicillin for 7 days CT scan of the sinuses Nasal decongestants and follow up Plain radiographs of the face

Correct Answer ( C ) Explanation: The patient presents with symptoms consistent with acute rhinosinusitis and should have a topical or systemic decongestant prescribed and follow up arranged. Approximately 1-2% of viral upper respiratory tract infections are complicated by rhinosinusitis. Symptoms often depend on the sinus affected. Sphenoid sinusitis may cause headache alone. Maxillary sinusitis causes pain in the area of the zygoma as well as maxillary dental pain. Ethmoid sinusitis can cause ocular or periorbital pain and frontal sinusitis can cause severe headaches around the orbits and forehead. The classic symptoms of rhinosinusitis are mucopurulent nasal discharge, facial pain, fullness over the sinus, nasal congestion and a pressure-like feeling. Acute rhinosinusitis usually builds in intensity over 7 to 10 days. The majority of acute rhinosinusitis cases are caused by viruses and will spontaneously resolve with supportive therapy alone. Bacterial causes are suggested when the patient has not improved after 7 days of conservative therapy or symptoms are worsening within 10 days of illness onset. Some patients will also experience "double-sickening" or an initial improvement followed by worsening symptoms. Supportive treatment should consist of analgesics, antipyretics and decongestants. A CT scan of the sinuses (B) may reveal air-fluid levels, mucosal thickening or sinus opacification but it should not be routinely performed as it is nonspecific (up to 40% of asymptomatic patients can have mucosal thickening on CT scan). CT should be reserved for those patients with significant symptoms. Amoxicillin (A) or alternatively amoxicillin-clavulanate are appropriate antibiotics for bacterial rhinosinusitis. They should be reserved for bacterial rhinosinusitis (described above). Plain radiographs of the face (D) have largely been replaced by CT scan when imaging is required as they are both insensitive and nonspecific.

A 65-year-old man is brought to the ED complaining of nausea for the last two hours. On arrival to the emergency department, he has the cardiac rhythm seen above. His blood pressure is 110/70 mm Hg. He denies any headache, chest pain, or difficulty breathing. Which of the following is the most appropriate next step in management? Defibrillation Magnesium sulfate Procainamide Synchronized cardioversion

Correct Answer ( C ) Explanation: The rhythm seen in the image is a monomorphic ventricular tachycardia. This occurs in most cases due to the presence of scar tissue in the myocardium. The first objective for treating any patient with a wide-complex tachycardia (WCT) is to evaluate for hemodynamic instability (hypotension, chest pain, dyspnea, altered mental status) because any patient with a WCT can deteriorate quickly as the rhythm degenerates into ventricular fibrillation. In stable patients, procainamide is the drug of choice and will terminate the rhythm in the vast majority of cases. It is superior to amiodarone and lidocaine. Not only is it useful in the termination of stable ventricular tachycardia, but it also blocks accessory pathway conduction, which terminates preexcited tachycardias. Sotalol, a class III antidysrhythmic with beta-blocker properties, can also be used in such patients though it is considered a second-line alternative. Defibrillation (A) is reserved for pulseless ventricular tachycardia and ventricular fibrillation. Magnesium sulfate (B) is the treatment of choice for polymorphic ventricular tachycardia (torsades de pointes). If medical management fails or the patient begins to decompensate, the patient should immediately receive synchronized cardioversion (D).

Which United States Food and Drug Administration category describes medications in which the safety of use by pregnant woman is unknown and the drug should not be used unless the potential benefit outweighs the potential risk to the fetus? Category A Category B Category C Category D

Correct Answer ( C ) Explanation: The safety of category C medications in human pregnancy is unknown. This is either because animal studies have shown fetal risk or have not been conducted. These medications should only be used when the potential benefits clearly outweigh the potential risks. Medications are considered to be category A (A) when well-controlled studies of pregnant women have failed to show any risk. Category B (B) medications are those in which animal studies have failed to show any risk to the fetus, but no well-controlled human studies exist. Medications in category D (D) have shown fetal risk in investigational or marketing experiences. The FDA has developed a classification system to categorize mediations based on potential teratogenic effects. Medications are assigned to one of the following categories based on human and animal data. [The FDA has recently updated these guidelines]

An 18-year-old college student with a history of HIV (CD4+ 250) presents to the ED with headache, fever, and stiff neck for two days. He thought he had a cold and has been taking acetaminophen without relief of his headache. Vital signs are T 39.1°C, BP 100/50 mm Hg, HR 140 bpm, RR 30. He is sleepy but arousable. On exam, you place the patient's right hip and knee into a flexed position and then proceed to extend the knee. The patient winces when the knee is just beyond 90 degrees of flexion. You also note petechiae on his trunk and extremities with one small area on his right forearm that looks like a purple patch with a gray necrotic center. Which of the following is the most likely diagnosis? Cryptococcal meningitis Herpes encephalitis Meningococcemia Pneumococcal meningitis Toxoplasmosis

Correct Answer ( C ) Explanation: This patient has meningococcemia, a disease caused by Neisseria meningitidis. The clinical presentation ranges from a mild febrile illness to fulminant disease progressing to death within hours. Patients with meningococcal meningitis may present similarly to patients with meningitis of other origins with headache, photophobia, vomiting, fever, and signs of meningeal inflammation. Petechiae generally appear on the extremities and may progress to involve almost any body surface. Macular lesions may progress to purpura and ecchymoses in fulminant meningococcemia (purpura fulminans). The patient in this scenario exhibits a positive Kernig sign, representing meningeal irritation, and has a purpuric lesion on his right forearm characterized by a gray necrotic center surrounded by a purple ring. Morbidity and mortality are high in meningococcemia but reduced with prompt recognition and immediate initiation of antibiotic therapy. Ceftriaxone and vancomycin are acceptable first-line agents. Patients with HIV are prone to the same infections as a non-HIV infected individual, in addition to opportunistic infections. The best predictor of immunologic susceptibility to opportunistic infection is the CD4 cell count, with counts below 200 associated with increased risk. Cryptococcal meningitis (A) occurs in 10% of patients with HIV infection, but most commonly in those with CD4 cell counts less than 100 cell/µL. Headache and fever commonly occur, but there are no cutaneous manifestations. Treatment includes amphotericin B plus 5-flucytosine. HSV encephalitis (B) is associated with fever, headache, and focal neurologic signs, often localized to the temporal lobe. The patient may complain of a bad odor not perceived by anyone else (temporal lobe hallucination). Treatment includes acyclovir. Pneumococcal meningitis (D) is caused by Streptococcus pneumoniae, the most common cause of meningitis in adults. Symptoms are similar to meningococcal meningitis but do not progress to purpura fulminans. Treatment is also similar with ceftriaxone and vancomycin. Toxoplasma Gondii (E) infection is the most common cause of focal intracranial mass lesions in patients with HIV infection. It is associated with fever, headache, and seizures. It is not associated with skin lesions. Treatment includes pyrimethamine plus sulfadiazine.

A 15-year-old G1P0 woman at 23 weeks presents with sharp, left lower quadrant abdominal pain for 1 hour. She has had an ultrasound confirming the presence of a single intrauterine pregnancy. The pain is severe and associated with nausea. Pelvic examination reveals tenderness of the left adnexa. The patient's urinalysis is unremarkable. What test should be ordered to diagnose the patient? Abdominal X-ray CT scan of the abdomen and pelvis Pelvic ultrasound White blood cell count

Correct Answer ( C ) Explanation: This patient presents with a history concerning for ovarian torsion and should undergo a pelvic ultrasound. Ovarian torsion is an organ threatening disease, involving twisting of the ovary or fallopian tube or both on the vascular pedicle. It is more commonly seen on the right side (due to the effects of the sigmoid colon being on the left) and in women of childbearing age. Patients typically present with sharp, severe unilateral abdominal pain with nausea or vomiting. However, the classic presentation is often not present making diagnosis challenging. Risk factors for torsion include the presence of an ovarian mass or infertility treatment. Clinicians must maintain a high suspicion for this disease in order to make a timely diagnosis and prevent ovarian necrosis. Pelvic ultrasound, while imperfect, represents the best initial imaging modality. The classic ultrasound appearance in torsion is enlargement with a heterogenous stroma and peripherally displaced follicles. The most common findings are an increased ovarian size and an abnormal position in relation to the uterus. The addition of Doppler ultrasound may demonstrate decreased blood flow to the ovary but these findings are inconsistent. Additionally, Doppler ultrasound may be completely normal in intermittent torsion. Ultimately, patients with suspected ovarian torsion may require laparoscopy to confirm or rule out torsion. Abdominal X-ray (A) does not aid in diagnosis of ovarian pathology. CT scan of the abdomen and pelvis (B) has a sensitivity that is similar to ultrasound but exposes the patient to the risks of radiation and IV contrast. A small percentage of patients with torsion will have an elevated white blood cell count (D) over 15,000 but this is not a reliable finding.

A 33-year-old woman comes to the clinic complaining of blurred vision, cough, and fatigue. Her blood pressure is 135/90 mm Hg and temperature is 38.6°C (101.5°F). She has no past medical history. Physical examination shows several subcutaneous nodules on both lower extremities. Ophthalmologic examination shows uveitis. Chest X-ray shows bilateral hilar adenopathy and reticular opacities. Laboratory studies show leukopenia, eosinophilia, and an elevated erythrocyte sedimentation rate. Which of the following is the most likely diagnosis? Asbestosis Cystic fibrosis Sarcoidosis Tuberculosis

Correct Answer ( C ) Explanation: Uveitis, bilateral hilar adenopathy, dry cough, subcutaneous nodules (eg, erythema nodosum), and an elevated erythrocyte sedimentation rate (ESR) most likely suggest sarcoidosis. Sarcoidosis is granulomatous disease that can form nodules in multiple organs (eg, skin, lungs). Patients typically present with fatigue, weight loss, arthritis, dry eyes, blurry vision, and respiratory symptoms (eg, cough, dyspnea). Management usually involves the use of corticosteroids (eg, prednisone). Bilateral hilar adenopathy, seen on chest x-ray, is a classic finding associated with sarcoidosis. Asbestosis (A) is not likely in this patient given the lack of previous exposure to asbestos. Additionally, asbestosis primarily presents with respiratory symptoms (eg, dyspnea, cough, digital clubbing); systemic findings are typically not seen. Cystic fibrosis (B) is not likely because these patients typically present in early childhood. Tuberculosis (D) is not likely, given the constellation of findings in this patient. TB would typically present with respiratory and constitutional symptoms (eg, fatigue, fever, night sweats).

An otherwise healthy patient presents with 4 months of constipation. Her physical exam is unremarkable. Other than educating her on increasing exercise, which of the following initial treatments would you recommend? Disimpaction Linaclotide Polyethylene glycol Psyllium

Correct Answer ( D ) Explanation: Constipation is one of the most common gastrointestinal symptoms in the United States. Common complaints include bloating, rectal pain or bleeding on defecation and low back pain. Rome III diagnostic criteria require 2 of the following symptoms lasting >3 months: fewer than 3 bowel movements per week, straining, lumpy or hard stools, sensation of anorectal obstruction or incomplete voiding or manual action required to defecate (digital extraction). Worrisome associated symptoms include any bleeding, abdominal pain, vomiting or inability to pass flatus. Further evaluation requires perineal and rectal examination, ruling out infective processes with complete blood counts and abdominal imaging, lower GI endoscopy, defecography and colonic transit studies. There are several constipation medications. Most outpatient constipation can be relieved with diet changes such as increasing fiber (psyllium or methylcellulose products) intake and decreasing milk, coffee, tea and alcohol intake. Also, increasing daily exercise can be helpful. Bulk formers, like psyllium, have shown to be very beneficial and cost-effective. If a patient fails this easy option, then one should consider adding or changing to a different class, like senna or, docusate or bisacodyl. Stool softeners like docusate and stimulant laxatives like senna or bisacodyl may be tried. Other options include mineral oil (offers rapid treatment for acute cases) and tegaserod (a prokinetic used in severe cases). Inpatient care, or severe refractory outpatient management, may require transrectal enemas or manual disimpaction. Disimpaction (A) is usually necessary for inpatient care of the elderly with significant comorbidities, those with spinal cord injury and those with traumatic brain injury. Linaclotide (B) stimulates intestinal fluid secretion and transit and can be used for chronic constipation refractory to other treatment options. It is not recommended as a first-line treatment. Polyethylene glycol (C) is usually reserved for large volume bowel washouts, such as in preparation for surgical or endoscopic procedures. It can be used to increase stool frequency and consistency in patients with chronic constipation but is not a first-line treatment.

A 16-year-old girl presents to the ED via ambulance for general pain. She is a refugee from a conflict area who is known to have frequent nighttime visits to the ED over the past year for the same chief complaint. She's undergone multiple medical workups that have all been negative. In the ED she appears angry, irritable, and demonstrates hypervigilance. After a brief conversation with the patient her pain resolves and she feels much better. Which of the following is the most likely diagnosis? Adjustment disorder Borderline personality disorder Malingering Posttraumatic stress disorder

Correct Answer ( D ) Explanation: Posttraumatic stress disorder (PTSD) is a long-lasting anxiety response following a traumatic or catastrophic event. Although most people encounter trauma over a lifetime, about 20-30% develop PTSD but over half of these people will recover without treatment. Prediction models have consistently found that childhood trauma, chronic adversity, and familial stressors increase risk for PTSD. Other risk factors include military experiences, war-zone exposure, domestic violence, and foster care. PTSD often leads to patients having difficulty falling or staying asleep, problems with concentration, hypervigilance, irritability, angry outbursts, and increased startle response. The patient in the above clinical scenario is a refugee from a conflict region and exhibits symptoms consistent with PTSD (anger, irritability, and hypervigilance). An important management principle when caring for a patient with PTSD is to ensure his or her safety and to validate his or her symptoms. Detailed questioning should be avoided as it may trigger severe symptoms. Borderline personality disorder (B) is characterized by unstable personal relationships, unstable self-image, and inappropriate behaviors. Patients with BPD usually present to the ED after deliberate self-injury or suicidal attempts. Malingering (C) is fabricating or exaggerating the symptoms of mental or physical disorders for secondary gain. This may include financial compensation, avoiding school, work or military service, obtaining drugs, getting lighter criminal sentences or to attract attention or sympathy. An adjustment disorder (A) occurs when an individual is unable to adjust to or cope with a particular stressor, like a major life event. The condition is different from anxiety disorder, which lacks the presence of a stressor, or posttraumatic stress disorder that is associated with a more intense stressor.

A young woman presents with bilateral headache. She describes it as a band like pressure from her forehead to neck, feeling as if her "head is in a vice." She denies prodromal symptoms, nausea and photophobia. Cervical spine range-of-motion and skull palpation do not reproduce the headache. Which of the following is the most likely diagnosis? Cluster headache Migraine without aura Occipital headache Tension-type headache

Correct Answer ( D ) Explanation: Tension-type headache (TTHA) is the most common type of recurring headache other than migraine and the most common type overall. Tension headaches present as a bilateral headache with pain in the frontal and occipital regions in a bandlike distribution. Headaches are exacerbated by stress, fatigue, glare, or noise. It also often involves contraction of the scalp and posterior neck muscles, with a normal neurologic exam. Treatment is with aspirin, NSAIDs, and ergotamines. Unilateral, throbbing headache with nausea and photophobia is much more indicative of migraine headache (B) These symptoms are absent in tension-type headaches. Occipital headaches (C), sometimes called occipital neuralgia, C2 neuralgia, C2 headaches, are characterized by sharp shooting electric-like occipital pain that radiates cephalad, retroorbitally or in the distribution of the greater and lesser occipital nerves. This pain typically worsens with upper cervical spine range-of-motion. A common exam finding of occipital headaches is reproduction of radiating pain during palpation of the occipital nerves as they exit the ligamentum nuchae at the base of the skull. Cluster headache (A) has a predilection for males and is associated with excrutiating periorbital pain, ipsilateral conjunctival injection, lacrimation, and nasal dscharge.

A 14-year-old boy developed an erythematous, papular rash over his palms and fingers. A complete physical examination reveals scaling and cracking in the interdigital webs of his feet. No other areas are affected. What is the most appropriate treatment? Intramuscular benzathine penicillin G Permethrin cream applied from the neck down Supportive care for hand-foot-and-mouth disease Terbinafine cream applied once daily to feet

Correct Answer ( D ) Explanation: The above patient presents with auto-eczematization, also known as an id reaction, which is thought to be due to a delayed-type hypersensitivity reaction to a fungal infection. In this case, it is secondary to his tinea pedis. Tinea pedis, also known as athlete's foot, is the most common dermatophytic fungal infection. Affected individuals develop vesicles or papules over the soles and interdigital webs of the feet, followed by scaling and cracking. It most commonly results in sterile vesicles over the palms and soles. The id reaction resolves with treatment of the primary infection. The treatment of choice for tinea pedis is a topical antifungal, such as terbinafine or clotrimazole, applied to the affected area for four weeks. The correct answer is terbinafine applied once daily to feet. Intramuscular benzathine penicillin G (A) is the treatment of choice for syphilis. Although the rash of secondary syphilis typically includes the palms and soles, it does not include cracking and scaling of the interdigital webs. The lesions caused by secondary syphilis vary widely in appearance and may be maculopapular, pustular, or annular. Lesions are typically erythematous or brown in color. Although less likely than an id reaction, syphilis should be considered in any adolescent with a rash of the palms and soles. Permethrin cream applied from the neck down (B) is the treatment of choice for scabies. While scabies does affect the interdigital webs of the hands, it does not cause scaling or cracking of the interdigital webs of the feet unless significantly excoriated. Moreover, scabies is intensely pruritic and spreads rapidly over the skin of the affected individual and also within families. Supportive care for hand-foot-and-mouth disease (C) is incorrect. While the child's hand lesions may resemble those of hand-foot-and-mouth disease, he lacks characteristic oral ulcers, rash on feet, or systemic symptoms of infection. Notably, if the patient did have hand-foot-and-mouth disease, supportive care, including pain control, would be the treatment of choice.

A postmenopausal woman presents with abnormal vaginal bleeding. You suspect endometrial malignancy, and refer the patient for further investigation. Which of the following modalities is initially used in determining a diagnosis? Colposcopy Dilation and curettage Papanicolaou test Suction curette

Correct Answer ( D ) Explanation: The most important condition to evaluate in a woman with postmenopausal pelvic bleeding is endometrial cancer. The cornerstone of diagnosis is based on endometrial sampling. Several modalities are available, but the easiest, most convenient and most cost-efficient process is accomplished in the outpatient setting through the use of an aspiration device, such as suction curette. Colposcopy (A) utilizes illumination and magnification of the vulva, vagina and cervix to evaluate the presence of premalignant and malignant lesions of these structures, but does not evaluate the endometrial cavity. Dilation and curettage (B) is reserved for patients with difficult or nondiagnostic outpatient biopsies. The results of a Papanicolaou test (pap smear) (C) may suggest endometrial abnormalities, but are unreliable in diagnosing endometrial cancer.

A 13-year-old boy presents to clinic with a worsening rash. The rash started on his right forearm and has progressed to involve the right upper arm and the left arm. It is not pruritic or painful. The rash has been present for about three weeks. He also complains of headache and fatigue. The patient returned from a camping trip in Pennsylvania one month ago. On exam, he has several erythematous macules with central clearing on the upper extremities and trunk. Which of the following associated findings is this patient most likely to have? Chronic encephalitis Complete heart block Monoarticular arthritis Peripheral facial nerve palsy

Correct Answer ( D ) Explanation: This patient has early disseminated Lyme disease as evidenced by the secondary erythema migrans lesions. Lyme disease is caused by transmission of Borrelia burgdorferi, a spirochete, via the bite of the tick Ixodes. Transmission is most common in the northeast United States. Lyme disease occurs in multiple stages. The initial stage is early localized disease occurring within the first month of exposure, characterized by erythema migrans at the site of the tick bite. Next, early disseminated disease may develop from release of the spirochetes into the bloodstream, three to twelve weeks following exposure. The classic findings in early disseminated disease are secondary erythema migrans lesions, constitutional symptoms, cranial neuropathy, meningitis, and carditis. The most common cranial neuropathy is facial nerve (VN VII) palsy. In endemic areas, Lyme disease is the leading cause of facial nerve palsy. Chronic encephalitis (A) is a finding in late Lyme disease. Symptoms include memory loss and cognitive deficits. These findings are rare in children. Complete heart block (B) is one form of carditis that may occur in early disseminated Lyme disease. However, the incidence of carditis is less frequent than the neurologic sequelae. Patients with suspected Lyme disease should have cardiac monitoring including an ECG. Arthritis (C) is the most common finding in late Lyme disease. This may occur months after the initial infection. The arthritis is usually monoarticular involving large joints. The arthritis may resolve after several weeks or may recur over time.

A 73-year-old man presents with vomiting and abdominal pain for 2 days. The patient has a remote history of cholecystectomy and appendectomy. Examination reveals a markedly distended abdomen and absent bowel sounds. Lab studies show an elevated WBC count and a lactate of 4.3 mmol/L. An abdominal radiograph is obtained that is shown above. Which of the following is the most appropriate management at this time? Administer intravenous fluids and obtain a CT of the abdomen and pelvis Arrange for emergent colonoscopy Perform a soapsuds enema and administer polyethylene glycol Place a nasogastric tube, begin antibiotics and obtain a surgical consultation

Correct Answer ( D ) Explanation: This patient presents with a high-grade small bowel obstruction (SBO) with evidence of bowel ischemia (elevated lactate). Mortality has fallen in the last century with aggressive surgical treatment (from 60% to 5%). The abdominal radiograph above shows multiple air-fluid levels consistent with an SBO. Radiographs are abnormal in 50-60% of cases and are more likely to demonstrate abnormality when the obstruction is high-grade versus partial. Two views (upright and supine or supine and decubitus) should be obtained. Mechanical obstruction refers to the presence of a physical barrier to the flow of intestinal contents. In a simple obstruction, the intestinal lumen is partially or completely obstructed causing intestinal distension proximally but does not cause compromise of the vascular supply. In a closed-loop obstruction, a segment of bowel is obstructed at two sequential sites usually by twisting on a hernia opening or adhesive band leading to compromise of blood flow eventually resulting in bowel ischemia. Ischemia may only be seen on CT scan or occasionally, on laparoscopy or laparotomy. However, an elevated lactate in the setting of an SBO is highly suggestive of intestinal ischemia. The presence of blood in stool (either gross blood or guaiac positive stools) also suggests the presence of ischemia or infarction. When compromise of the vascular supply is suspected, the patient should have an emergent surgical consultation for operative management. Immediate management should also include placement of a nasogastric tube for decompression of the proximal parts of the intestines, intravascular volume resuscitation and intravenous antibiotics when vascular compromise is suspected or confirmed. CT scan of the abdomen and pelvis (A) is considered complimentary to plain films and is more sensitive and specific. Additionally, CT scan can reveal the site and cause of obstruction. However, surgical evaluation of a high-grade SBO should not be delayed for advanced imaging. Colonoscopy (B) is not indicated in small bowel obstruction. There is an increased risk of perforation. An enema and polyethylene glycol (C) is the treatment for constipation, and may worsen the outcome in patients with high-grade bowel obstruction.

A 55-year-old woman presents to your office with complaints of fatigue, dry skin, constipation and weight gain. Physical exam results include a heart rate of 58 bpm and diminished deep tendon reflexes. Laboratory testing reveals an elevated TSH. Which of the following is the most appropriate next step in management? Begin treating patient with a beta-blocker Begin treating patient with levothyroxine Test serum free T3 Test serum free T4

Correct Answer ( D ) Explanation: When a patient presents with symptoms of hypothyroidism, serum TSH is the initial test indicated. An elevated TSH alone does not provide enough information to diagnose hypothyroidism. Other pathologies may cause elevated TSH such as resistance to TSH or thyroid hormone, a TSH-secreting pituitary adenoma, or primary adrenal insufficiency. When TSH is elevated, serum free T4 should then be tested. High serum TSH with low serum free T4 indicates primary hypothyroidism. Clinical signs and symptoms of hypothyroidism can vary greatly, and diagnosis is reliant on laboratory testing. Treatment for primary hypothyroidism is replacement therapy with T4. Beta-blockers (A) are used to treat symptoms of hyperthyroidism and have no role with hypothyroidism. Treatment with levothyroxine (B) should begin once serum free T4 is evaluated and primary hypothyroidism diagnosed. Serum T3 (C) is tested when hyperthyroidism is suspected based on clinical symptoms. Patients with hyperthyroidism caused by Graves' disease will have a higher serum T3 level than serum T4.


Ensembles d'études connexes

BCH480 All quizzes/Test Questions

View Set

Chapters 4-5 Learn Smart; Biomechanics

View Set

Virginia Life & Health Study Guide #1

View Set

FINA final conceptual questions from all

View Set

ch two: beginnings of english america

View Set

Real Bis: VA Law and Regulations - Chapter 5

View Set

UCCS Basic Health Assessment Chap. 10

View Set

Economics Ch.4 Market Equilibrium

View Set

Unit 6 Cirrhosis, Lupus, Kidney Failure, Menopause

View Set